UP | HOME

Georgism & Natural Resources Taxation FAQs

1. Introduction (Start Here)

This FAQs is a work in progress. It was created by compiling the following FAQs pages, and adding my own (Zero Contradictions) contributions and edits to the FAQs.

Readers may be interested in Mark Wadsworth’s Killer Arguments Against LVT… Not. Is probably the most comprehensive on the entire Internet, even moreso than this one. However, I dislike its CSS, some of the wordings, and many of the arguments that it makes or prioritizes when arguing for Georgism. It is remarkably thorough, but it could still be greatly improved. Unfortunately Wadsworth is deceased (RIP), but if I have the time, this FAQs may possibly someday outshine his impressive work. Perhaps some AI could even generate something that’s even better with the best of both worlds.

Many of the questions on this page address the same objections and misunderstandings, so it may feel repetitive and redundant to read all the questions and answers on this page. If you would like to read an actual essay explaining what Georgism is, then we recommend reading: Georgism Crash Course.

Since this FAQs page aims to be the ultimate Georgist FAQs page on the entire Internet, it’s still a work in progress. Questions that have been taken from sections of other FAQs files, but whose content has not yet been modified to be properly formatted and written in the same concise post-overton style as everything else on this website are marked with “-N” at the end of the question. Eventually, they will be copy-edited to give a consistent feel and the most concise information dense style as the rest of the FAQs.

Disclaimer: I haven’t read the book yet, but Critics of Henry George is said to have a very comprehensive documentation and analysis of all the major critics and criticisms of Georgism. It may have many important ideas and arguments that are not addressed on this page.

2. Answering Questions About The Implementation, Logistics, And Consequences Of Georgism

Draft Georgist taxation plan with as many details as possible regarding:

  • Describe the implementation of Georgism in as much detail as possible.
  • Give more specific details regarding the implementation of taxation rates across different types of land and varying land values.
  • How would the mathematics (and machine learning?) of the mass appraisal be calculated?
  • How much taxes would be collected, and how often?
  • How would the various land value taxes be adjusted for inflation?
  • Who would get taxed when the ownership title isn’t very clear?
  • What would be the detailed route of procedure if someone does not pay their LVT?
  • When land is available for people to settle it, would they be signing a contract with the government to pay the designated LVT rates?
  • If someone does not pay their LVT to the point where they forfeit their occupation of the land, do they lose the improvements on the land, do they get compensated for losing those improvements, and/or does the value of those improvements get used to help pay the LVT that they failed to pay?
  • When land is available for people to settle it, would someone have to buy the improvements on the land as well?
  • What are the most realistic reactions to the gradual implementation of Georgism?
  • What are the most realistic reactions to the Georgist taxation system itself?
  • How would the logistics of the collection of other economic rents (besides real estate values) work?
  • How would pollution be measured before collecting pollution taxes? would it be possible to measure all types of pollution?
  • How would prices be effected regarding: food grown on farms, labor, etc?
  • Can it be verified that rents cannot get passed onto the tenants, when taking into account the effects of capital markets?
    • Although this page claims to prove that LVT cannot get passed onto the tenants, it’s actually not very helpful because it doesn’t cover the effects of capital markets at all, and the empirical study discussed in the article was on tax rates too low to cause an exodus of capital from real estate. As far as I know, nobody has taken this into account when arguing that LVT cannot be passed onto the tenants.

2.1. How would such a system be implemented? -N

In short, the same way it is now. Critics of the LVT are fond of pretending that land values are not already being taxed, when in fact they are (albeit to a limited extent) by existing property taxes. The machinery for the LVT is already in place. Thus, all that is necessary to implement the LVT locally is to exempt houses, buildings and other improvements from taxation, and thereby focus existing property taxes on land values only. In this way the property tax would be converted to a land value tax. As for state and federal taxation, geolibertarians advocate a bottom-up system whereby a portion of the LVT-revenue generated locally is sent to the applicable state governments, and a portion of that, in turn, to the federal government. Ideally, this would be phased in over a period of years. That is, as the LVT is slightly increased each year, taxes on wages, sales and capital goods would be slightly decreased. This process would continue until all taxation is eliminated save for a single tax on land values.

Whether a political party were elected on a site rent platform, or a government were persuaded to change over in mid term as the result of a referendum, then there would obviously need to be a period of transition in which legal matters were sorted out, legislative changes made, procedures established etc.. This could possibly take a year or so. At the point where the system becomes law, then the current unimproved site values at that time (as is already available for scrutiny wherever rates are collected) would be used as the basis on which the first annual rental is struck at a rate of say 10%, or whatever was determined as required for government expenditure (using the same methodology to set rates).

Although initially set at ten percent on the unimproved value of all sites, the asking price for land would obviously fall dramatically on the introduction of the site rent system, and would drop in many areas to zero as the supply of sites outstripped demand. Site Rental, representing Land Value, replaces the inflated Land Price that is possible at present with wasteful land usage. At this point, within a year or so of the implementation of the scheme, the government would simply collect the annual site rent as its sole source of revenue. In other words, the occupier would enjoy exclusive use of the site for as long as they continue to reimburse the community for the benefits which their exclusive access gives them. In that way, the value created by the community’s presence – the economic rent – is returned to the community as revenue for its government.

There would no doubt be a bit of panic, especially among all those whose whole or partial income derives from rent in one form or another, but even these people would see, once they’d calmed down, that the benefits to the whole economy of such a radical change would massively outweigh any losses. Those whose income though is largely gained from community generated rent through monopolistic holdings of land and resources, licenses, etc. would be violently opposed, obviously, as their livelihood would be threatened and they would be reduced to relying on their own skill and efforts to produce income. They would be forced to live on their earnings and not their incomes.

On the positive side for these people would be the fact that they would still retain their mansions and their yachts; the legitimate part of their income from shares in productive enterprises like BHP, CRA etc., would still be coming in – in fact it may increase, as these enterprises would be taking part in a massive resurgence of industrial and corporate activity fueled by higher wages in the pockets of the consumers and cheaper manufacturing costs for the exporters. So even they would not necessarily be any worse off in the end, and would enjoy living in a happier, less fear-ridden and corrupt society. The best salve to their fears would be to give them all a years supply of valium, and a government-subsidised short course along the lines of “Only work generates wealth”.

Regarding the immediate redundancy of the very large numbers of taxation and related department functionaries, including Ministers and Ministerial departments ; even if the government undertook to continue employing them to twiddle their thumbs rather than paying out large packages, I think most of these people, within months of the start of the new system would be eager to get into it, seeing that they could employ their talents in a totally free market and generate as much wealth as they wanted to.

3. Switching to Georgism

Any conversion over to a Georgist taxation system would have to be a very gradual process, taking at least 30 years in order to give everybody enough time to re-adjust their personal finances, especially for the people who are relying on land speculation as part of their retirement portfolio. But that’s really the only drawback to Georgism. Once you’re through that, it’s smooth cruising from there on out.

The way how I was willing to switch from Qwerty to Colemak, in spite of the 1-month transition period, is analogous to how I support Georgism, which would also require a transition period. Both provide better long-term results because they are both applications of the Space Utility Optimization Principle.

Additionally, in order for Land Value Tax to be truly effective, it must be widespread, otherwise those not having to pay a land value tax will have an even bigger advantage. This is just like how states without income tax have an advantage in labor costs over states that do have income taxes, or how states without sales taxes have an advantage in sales over states that do have sales taxes. Likewise, if a Georgist government imposes taxes on negative externalities, like pollution, there would need to be a tariff on goods from countries which didn’t impose such a tax. Otherwise you would just be subsidizing pollution elsewhere.

It is already common practice in the real estate industry to separately evaluate of land values from property values. The easiest way to transition to Georgism would be to gradually transition the Property Taxes so that they tax the unimproved value of the land instead of the improved value of the land, in increasingly greater proportions over time. Property Taxes are basically a combination of a good tax (land) with a bad tax (property), so you slowly reduce the bad tax proportion to nothing while increasing the good tax proportion (LVT) to higher amounts. And then from there, you slowly reduce sales taxes, income taxes, and so forth to 0%, while slowly increasing the LVT to 100%. We already have all the machinery for implementing Georgism, it’s just a matter of gaining enough political support for starting the decades-long transitioning phase by educating more people about economics.

4. Why are natural resources taxes better than other taxes?

Most existing forms of taxation punish productivity. Most existing forms of taxation are complex, ad hoc and create opportunities for corruption and tax evasion.

Although natural resource taxation has some complexity, it is much simpler than existing forms of taxation. There are a relatively small number of physical inputs to production, and they are relatively easy to audit and control. Natural resource taxation would be much simpler and fairer than applying taxes to every economic transaction, which is how the current system works.

4.1. Why would Georgism tax natural resources, and not just land?

By taxing the economic inputs of the economy instead of the economic outputs, there would be incentives to use the inputs (Natural Resources) more efficiently, and we should absolutely want this since there is a fixed, finite amount of valuable land and natural resources on Earth. Likewise, people wouldn’t get penalized for generating greater economic outputs.

Although location value taxation would be the main most dominant form of taxation, the taxation of the following would depend on how large the demand for those resources is, relative to the supply:

  • Mineral Deposits
  • Forests
  • Fish Stocks
  • Geostationary Orbits
  • The Frequencies of the Electromagnetic Spectrum
  • Reproduction Licenses

The economics is pretty easy to understand, provided that one has an understanding of supply and demand.

4.2. Why are natural resource taxes better than a progressive income tax?

Income tax is based on income earned by labor or investment. If you work hard and well, or invest wisely, you will pay more income tax. This punishes productivity, and it makes labor more expensive. Income Tax is the sort of thing you would come up with if you were specifically trying to make the economy less efficient.

Income tax also requires individuals and companies to be accountants and keep track of financial minutia — or lie about them.

4.3. Why are natural resource taxes better than a flat income tax?

A simple flat tax would be relatively easy to assess compared to progressive income tax, but in most countries the income tax is very complex, with many loopholes and subsidies.

A flat income tax may have less harmful effects than a progressive income tax, but it would still punish productivity, and it would make labor more expensive nonetheless.

4.4. Why is natural resource taxes better than value added tax / sales tax?

VAT would reduce economic activity by decreasing sales in the economy. Consumption taxes also punish productivity, although not directly. All income is eventually used for consumption of some kind. So, a tax on consumption is a tax on income, and thus a tax on production.

Value-added tax is very complicated, because it requires a full accounting of revenue and the costs of production. Sales and value-added taxes are often ad hoc, and vary from one jurisdiction to another.

Sales tax requires businesses to record every sale, so it incentivizes them to not record every sale. By comparison, it is way easier to hide purchases and transactions than it is to hide land, especially since those purchases and transactions would be private knowledge, but not public knowledge (unless you make an authoritarian law mandating that everybody’s transactions be made public, but even then, people would still find a way around that). This is yet another reason why VAT is more prone to tax evasion, and would require significantly more paperwork than LVT.

More Information: Why FairTax is a Bad Idea.

4.5. Why should we implement land value taxes when we already have property taxes?

Property Taxes make housing less affordable, whereas Land Value Taxes make more affordable. Property Taxes reduce the supply of housing (by making it more expensive to build housing), and since the tenants thus have fewer options/alternatives to choose from, this enables the landlords to pass the Property Taxes onto the tenants. This wouldn’t happen at all with LVT though, because LVT would incentivize landlords to build more housing per square foot and they wouldn’t get penalized for creating new floors on their land. Since LVT would increase the supply of housing, the tenants would have many options to choose from in this buyers’ market, so LVT would not get passed onto the tenants.

People who own buildings wouldn’t be disincentivized to not upgrade them anymore. If property taxes are abolished, then people won’t get penalized for renovating their properties.

Property taxes also tax economic improvements to the land, which disincentivizes people to renovate their properties and creates the aesthetics similar to urban decay. Taxing economic improvements (which require human labor) makes the economy less efficient because that would effectively be a tax on productivity, and thus higher undesirable.

Furthermore, property taxes don’t bootstrap prices within the economy, nor do they conserve natural resources.

4.6. Why don’t we impose reciprocal import duties? -N

This solution and any other which makes goods more expensive to the local consumer simply reduces the market for the imported product, and increases the price of the home grown substitute. The only winner there is the producer of the protected product, who, with less competition can boost his price without having to worry so much about service or quality. He has a captive market. Tariffs and duties punish the consumer by inflating prices and reducing choice and quality.

5. General Questions

5.1. What do Georgists think about Private Property?

If I make something with someone else, do they have just as much ownership over it as me?

That’s a game theory problem. The workers need to come to some agreement about how much value they each deserve from making the product.

How does one lend enough land for everyone to get access to it?

The state don’t lend land to everyone, because not everybody needs to possess land in order to work.

Is it realistic that every citizen of a country can rent land?

Not really.

Are there more than one renter for each land plot?

There can be. It would depend on who is assigned to the land title.

5.2. Why should landowners pay land value tax?

  • There is an interesting split between geoists who believe those who create land value are obligated to receive it and geoists who believe those who occupy land value are obligated to return it.
  • Private communities, under geoism, are glorified land owners. They operate functionally indistinguishable from a residential landlord/homeowner. The only difference would be scale.
  • This means such communities are obligated to return the land value their community resides on to everyone outside the community.
  • This would seem an insurmountable task given the size of private communities, but it is made easier if the residents of the communities themselves are ultimately paying LVT for each subsection they reside on.
  • This means the private community organizer is off the hook for the majority of the land value.
  • In fact, they might ultimately get paid if their jurisdiction over said community makes living in that community more valuable than otherwise. This is the Henry George Theorem in action.
  • Knowledge that all other entities in a community are obliged to follow a minimal set of rules (other rules might be built on top of these selectively) and that one would receive certain community services from the private community organizer might make the community a profitable enterprise.
  • It is also easier to resolve the problem of rent because one doesn’t need to track down the source of rent, only its existence and magnitude. This problem by itself is problematic enough. Who is obligated to receive rents is another problem I haven’t figured out.

5.3. The supply of land offered for sale in the market often changes — so why do Georgists say that land is a monopoly? -N

Yes, the quantities offered for sale are not fixed, but the total amount of land available is fixed. The sale of land just changes the persons who have title. The total quantity is important in setting the market rent and price of land. There are no competing suppliers for any particular land site. The fixed total quantity, and the fact that land was provided by nature, makes land rent an economic surplus that can be tapped with no economic damage.

5.4. Could rents get passed onto the tenants? -N

No. There is a consensus among economists that Land Value Tax cannot be passed onto the tenants.

The basic gist is that tenants can go to another location with a lower location value. This property owner here is being charged less in taxes because the location value is lower. That would just mean they pass on slightly less expenses to their tenants, right? It would, except the tenants can keep going to locations with lower and lower location value until they reach the location with zero location value. The land here is essentially worthless. They would prefer to live closer to the city but their expenses for living here, further out, and the expenses they would incur for living closer in the form of higher rent are identical. The landlord must provide superior service rather than charging more for increase in location value.

Site rent cannot be passed onto the tenant because the landlord is already charging the market price. Additionally, it cannot be passed on because there is now an increased supply of rental properties on the market. Tenants can now threaten to move if the landlord attempts to pass it on. When rent is collected on unimproved values, land will become much more readily and cheaply available. Many more people will have access to sites and be in a position to build their own houses and create or operate businesses. All you’d need to borrow, if at all, would be equal to about 10% of the current unimproved site value; enough to cover the first year’s site rent. Compare this to the gigantic burden of current mortgages at inflated interest rates. Many existing tenants would move and take up sites elsewhere. The law of supply and demand will reverse the current situation by forcing landlords to compete to attract tenants by: improving terms and conditions, carrying out building and internal improvements, undertaking promotional activities in commercial centres, etc. In other words they would have to start renting buildings alone and forego the unearned income from the site itself. In such a market, if your landlord tried to increase the rent, you would simply move. You could confidently threaten your landlord that if he didn’t lift his game, you would move out. In such a climate, landlords will be improving their buildings in an effort to attract clients. It will be a buyer’s market. Landlords, especially smaller landlords, would ultimately benefit from Community Site Rent in any case, in that, although they would be foregoing the site rent as part of their income, they would be in the same position as other citizens in that the benefits from a move to Community Site Rent would offset and probably outweigh their initial loss. Larger landlords, the greater part of whose incomes currently derive from ground rents alone, would probably be more likely to lose overall, and may well take their capital offshore. The community would probably gain from such an outcome. Readership of online independent media would surge during such a period of transition!

5.5. How would Georgism make housing more affordable?

5.6. What is the difference between Land Value and Land Price? -N

Land Price is the accumulated capitalisation of economic rent. It’s what the market pays in a marketplace where land speculation is encouraged and title is bought in a one-off transaction (as per at auction). Land value is the actual rental value of a site ie what can be physically earnt from that location.

McDonald’s is the world’s 2nd biggest landowner. They are renowned for ‘doing the sums’ before committing to a future store and buying a piece of land. They hire statisticians to count the number of cars that pass by the location. Using their statistical models, they calculate for example, that if 10,000 cars pass each day with a likelihood of 200 potential customers spending $8 each, the site is worth $11,648,000 ($1600 x 7 days x 52 weeks x 20 years). They will refuse to pay above that market price because they know they can’t earn enough to pay the mortgage. Under our system, they would refuse to pay more than $582,400 (1600 x 7 x 52 weeks) for a yearly site rental because they know the actual Land Value. To pay anything above this is uneconomical as they wont be able to earn the extra revenue to cover this. A similar situation is the astute small business investor who sits in for a month on a prospective business , recording all transactions (in say a Milk Bar) to see how much the business is actually worth, before committing to the asking price. Often under our present system, speculation forces Land Prices above what can realistically be earned by its occupants. Under a Site Rental system, when there are 70,000 cars passing by and 300 people entering the store, the company will naturally pay more back to the community, keeping in time with the growth of society. In time Land Price should equal zero, but Site Rental will replace that, growing as society does, ensuring the people get a share of the improvements.

5.6.1. What would happen to Land Prices?

The speculative component of land prices would be removed as the increased supply of property (huge tracts are withheld from supply by speculators) leads prospective buyers to pay only what the property is worth in terms of location, infrastructure and amenities. It’s probable that land prices would drop to zero within the first couple of years. Remember, there is a big difference between land price and land value.On the introduction of the system, anyone holding land speculatively would either have to put it to productive use or get rid of it. If they hung onto it, they would be out of pocket by the annual rental value. Putting it to productive use would result in a dramatic increase in demand for labor and therefore an increase in trade and thus in the general welfare. Getting rid of it would result in the asking price for land dropping substantially to the point where supply exceeded demand, with the seller getting progressively more desperate to sell and thus avoid the site rent.

In the end, land price is replaced by land value, as represented by the Site Rental. The Site Rental is representative of only what can be earned by occupying that piece of the planet.

5.6.2. Wouldn’t the land value tax (LVT) increase the price of land?

No, because it wouldn’t change the supply or the demand for land.

Most people who ask this question mistakenly think that the LVT would be paid on top of the price of buying land. But this is inaccurate because land would not be bought at all under Georgism, as land could only rented by paying the LVT (the price of the land rent).

Since the land rent is the only cost to being able to possess/occupy land, since LVT would effectively be the price of land, and since the price of land would only be dependent on the supply and demand of land, LVT would not increase the price of land.

5.7. But isn’t land speculation the same as any other commodity speculation?

[I haven’t finished writing this section yet. It takes time to write stuff.]

5.8. Why is only land speculation bad? Why not also SBF-style speculation?

Every form of rent-seeking is undesirable for an economy and that there should be laws to prevent it whenever it occurs. We oppose Sam Bankman-Fried style speculation, but it’s harder to make a case that that counts as rent-seeking. SBF-style speculation is more akin to gambling instead of investing since the speculators don’t really own any true wealth and the risks are much higher. Gambling in small amounts won’t hurt an economy, but it becomes a problem when it’s done on the scale of the FTX scandal and a feedback loop is created.

If the stock market counted as rent, then that would effectively mean that there is no value in researching to distinguish between profitable investments and non-profitable investments. To an extent, the same could be said for land speculation, but the key difference between speculating on land versus speculating on a stock is that the former deprives everyone else of a natural resource in fixed supply, whereas the latter does not.

One of the problems with making a law against SBF-style speculation is that it can difficult to define and differentiate. It may just be better to let gamblers suffer the consequences of their actions. I oppose capital gains tax because stocks represents real wealth, are actual investments in a company, and actually have positive benefits on the economy.

5.9. How can you predict that Georgism would increase economic growth?

One of the reasons why economics is so unpredictable is because economists have been assuming for centuries that social mobility is mostly dependent on environmental factors, when it’s actually dependent on genetics. The mathematical optimization behind Georgism is very sound. Another thing is that virtually all tax proposals have largely failed to benefit society in the past because they don’t aim to reduce rent-seeking, they punish productivity in one form or another. Neither applies to LVT.

But it’s based on assumptions that space is effectively utilized. Is that mere speculation?

There are many examples in that section. Just imagine how slow and inefficient your computer would run if it used a finder’s keepers approach to memory and CPU power. That’s the system that we have now (a finder’s keeper’s approach to land ownership).

I’m arguing for a system that allocates resources in the most efficient manner possible, similarly to how modern OSs are designed.

But a society is not based on processors and chips, it’s based on the unpredictability of humanity?

There’s a lot of complexity and unpredictability in ranking processes too. There are entire CS courses that teach memory, CPU, and resource management, like Theory of Computation.

Yes, but then the code isn’t even finished itself, so how would it be able to be successfully applied to humanity when it in itself isn’t completed in it’s own area? That would be very speculative?

Georgism works because it uses free market principles. We can’t know how valuable land is we don’t put a price on its monthly value. The same cannot be said of the current system, where land values are not appraised on a regular basis, since people can own land for years and decades, no matter how inefficiently they may be using it.

I think this is missing a perspective that the free market itself is what determines value, not some kind of appraisal system, and it seems unfair to say that a free market is good for everything but land.

That’s not true for natural resources.

5.10. How would Georgism resist government corruption?

[I haven’t finished writing this section yet. It takes time to write stuff.]

5.11. What other effects could Georgism have on the economy?

Georgism would increase fertility rates since it increases the society’s wealth.

[I haven’t finished writing this section yet. It takes time to write stuff.]

5.12. Why is income from land ‘unearned’ income? -N

Some say it’s an investment, and the investors deserve a return, just like someone who has bought shares or has money in the bank. A return of a certain percentage from investments in productive activities is fine. After all the investor has provided risk capital to some enterprise which is going to produce some product for public consumption, and the returns to the investor simply reflect the level of risk and an inflation factor. We have no problems with that. But if you buy a site this year, be it in the CBD or the outer suburbs, and hang on to it for ten years waiting for community funded infrastructure and pressure of artificially created land shortage, or rezoning to push up the price, then such profits as may arise in excess of those due to inflation, belong to the community. How could anyone dispute this?

5.13. There is no such thing as a free lunch. -N

The costs of Community Site Rent would be carried by every home owner, everyone who rents, and everyone who consumes goods or services which have land as an input. There shouldn’t be such a thing as a free lunch, but there certainly is. The wealth which is generated by the very presence of the community, and which ends up in private hands pays for many a free lunch, in fact many a free banquet with all the trimmings and has done for hundreds of years. These people would be the big losers, in that whatever part of their income derived from the community-created or common wealth would be absent from their incomes. This is not to say that J.Packer and his mob may still not generate a very respectable income from the legitimate employment of their undoubted directorial and entrepreneurial skills. All they would be foregoing would be their former share of the earnings of their fellow human beings. All of those among the wealthy elites in this lovely country of ours and elsewhere, whose incomes exceed their earnings are in effect stealing the bread from the tables of the millions whose earnings vastly exceed their income.

5.14. There is no unimproved land value! The only income generated from land is from the application of labor and capital to the land. -N

No, there is an unimproved value. It is the value put on land by the community before its even touched by the hand of man or woman. It’s the amount you would pay for virgin farmland, or an empty city site. It is this value, the unimproved value, which is the common wealth and which belongs to all in the community. Your improvements on the other hand belong to you. Every single drop of sweat, or byte of brain power you apply to the increase in value of the asset, be it a new building, a hotel, or a thousand hectares of wheat should be yours. Not the government’s. You make it, you keep it. But return the community created rent to the community.

5.15. Politically, is Georgism more of a left-wing or a right-wing ideology?

Part of the beauty of Georgism is that it can be compatible with nearly every political ideology there is. Georgism promotes equal land rights and free markets, both of which are common liberal values. There are geo-socialists, geo-conservatives, geolibertarians, and geo-anarchists. There is also no reason why Georgism can’t be compatible with a Christian/Muslim Theocracy, Monarchy, Communism, or even Fascism.

Some people have argued that the Georgist values in equal rights and more equitable land/wealth distributions makes Georgism more of a left-of-center ideology, but there’s no reason to categorize Georgism as being “inherently leftist”. Historically, all the earliest proponents for land value taxation were classical liberal economists and political philosophers. Today, those historical figures would be better categorized as Libertarians, not Leftists. Karl Marx was a very harsh critic of Henry George, and many leftists are also in favor of income taxes and/or sales taxes because they don’t believe that land value taxes would be enough to achieve wealth equality. So, it doesn’t follow that Georgism is an inherently leftist position, although you could argue that it does a better job at achieving leftist values than most leftists’ political proposals. If anything, it makes more sense to simply describe that Georgism is a liberal value. There are many different types of liberals who support Georgism, and it certainly wouldn’t be accurate to group them all together as “leftist”.

Georgism also has good synergy with many religious worldviews. Many Christians and Muslims believe that God created the Earth to be shared by all humans equally, so there are strong religious arguments for Georgism.

Fascists tend to favor mixed economies that aren’t perfectly “capitalist” or perfectly “socialist”. In a sense, a Georgist economy would already be a mixed economy that fits this description. So, Georgism could be compatible with Fascism too.

Georgism is compatible with a monarchy too. Under a Georgist monarchy, all land would be considered public property (nominally, the property of the King), while it is rented to private individuals.

In my personal experience, Alloidal Libertarians and Anarcho-Capitalists tend to be the most die-hard anti-Georgists out there. Ancaps are opposed to all taxes of every kind, and they even go as far as to pejoratively label Georgists as “Land Communists”. Ancaps very strongly value private property and private ownership. Nevertheless, there are some Ancaps who accept Georgism, but they’re usually gate-kept from being labeled as Ancaps and usually prefer to call themselves geo-anarchists to differentiate themselves. ZC’s Case Against Libertarianism page also has many great arguments as to why True Libertarians ought to be Georgist.

Above all however, I personally argue that Georgism should be characterized as a Pragmatopian and Neo-Malthusian position. Georgism and Neo-Malthusianism both aim to conserve natural resources that exist in fixed supply. Most Georgists tend to be Cornucopians since Henry George was a harsh critic of Malthusianism, but I’ve written extensively about Neo-Malthusianism and Georgism should work harmoniously together, rather than antagonistically. Georgism will eventually fail without some form of population control, so Pragmatopians are arguably the most Georgist of them all since we are the only ones who advocate for a sustainable Georgist political system that’s guaranteed to potentially last for centuries.

6. Wealth Distribution, Ownership, And Equality Questions

6.1. How does Georgism achieve “equal ownership of land”?

All land rents are used to pay for government revenues. Most Georgists propose a Citizen’s Dividend (nowadays known as UBI) if there’s a surplus in government revenue, but there are other alternatives to this.

6.2. Would the government be the de facto of all the Earth’s land? -N

[I haven’t finished writing this section yet. It takes time to write stuff.]

In a sense, yes. However, eeeeeeeeeee.

No, because government would have no authority to dictate when, how, or by whom land itself is used; it would only have the authority to ensure the rent of land goes to everyone on an equal basis, since all individuals have an equal right to the use of land. Henry George put it thusly:

“We do not propose to assert equal rights to land by keeping land common, letting any one use any part of it at any time. We do not propose the task, impossible in the present day of society, of dividing land in equal shares; still less the yet more impossible task of keeping it so divided.

“We propose–leaving land in the private possession of individuals, with full liberty on their part to give, sell or bequeath it–simply to levy on it for public uses a tax that shall equal the annual value of the land itself, irrespective of the use made of it or the improvements on it….We would accompany this tax on land values with the repeal of all taxes now levied on the products and processes of industry–which taxes, since they take from the earnings of labor, we hold to be infringements of the right of property.” [Emphasis mine] – The Condition of Labor, p. 8

The only alternative to George’s proposal is to treat land as the unconditional property of a relative few. The problem with this alternative is that, when taken to its logical conclusion, we find that the fruits of individual labor must inevitably be treated as conditional property for everyone else. Why? Because no one can produce wealth in the first place unless he or she first has access to land. Consequently, since all land is legally occupied, and since producing more land isn’t an option, those who don’t have titles to land cannot legally access the earth – and thus cannot legally sustain their own lives – unless they first “consent” to pay a portion of their earnings to those who do have titles to land. (This is why geolibertarians regard landed property as the mother of all entitlements.)

Land itself does not originate from labor; thus, property in land does not originate from labor, but from the law that confers ownership to an individual or group. Landed property is therefore law-made property, and is, in that sense, clearly distinct from man-made property. Thus, to compel one group to pay rent to another group for mere access to the earth is to elevate law-made property above man-made property. And since the latter is an extension of self-ownership, to elevate the former above the latter is to strike a blow at the very foundation of property rights.

“Disregard of the equal right to land necessarily involves violations of the unequal right to wealth.” – Max Hirsch, Democracy vs. Socialism, p. 372

To this some might object that the LVT does just that – compels one group to pay rent to another group for mere access to the earth. While this objection may sound logical at first, it is fatally flawed. Why? Because it ignores a universal law of today’s economy: the fact that land rent gets paid either way – regardless of whether or not it gets diverted into the public treasury.

Thus, it is not a question of if land rent gets paid, but to whom and on what basis.

If it is paid exclusively to titleholders on the basis of the earth being the unconditional property of titleholders, then, for reasons given above, the property that non-titleholders have in themselves and in the fruits of their labor is thereby violated. If, on the other hand, it is paid to the community on the basis of the individual members of that community each having an equal right to land, then said property right (the right to one’s self and the fruit of one’s labor) is thereby upheld foreveryone – both titleholder and non-titleholder alike.

Another common objection is that, if government collects the rent of land, it automatically becomes the owner of land. This objection is based on the myth that the terms “rent collector” and “owner” are synonymous. While many rent collectors do, indeed, own the property on which they collect rent, there are, nevertheless, thousands of private rental agents and property managers all over the country who routinely collect rent on properties they do not own. Thus, one does not have to be an “owner” to be a “rent collector.” Government is no exception to this rule.

That doesn’t mean the government of, say, North Korea does not assert ownership over the land on which it collects rent. It does. But it is not merely the authority to collect land rent, but the authority to dictate how land is used, that makes the North Korean government an “owner” of land. Critics of the LVT repeatedly insist that you can’t have one authority without the other, but as mentioned above, the rent-collection services provided by non-owning rental agents and property managers prove just the opposite.

This becomes easier to understand once you realize that “property” refers, not to a single right, but to a bundle of rights – the right to rental income being one of them. The other rights include the right to possess, use, exclude, and transfer title. As any lawyer will tell you, those rights can be transferred in whole or in part.

“The concept of a bundle of rights comes from old English law. In the middle ages, a seller transferred property by giving the purchaser a handful of earth or a bundle of bound sticks from a tree on the property. The purchaser, who accepted the bundle, then owned the tree from which the sticks came and the land to which the tree was attached. Because the rights of ownership (like the sticks) can be separated and individually transferred, the sticks became symbolic of those rights.” [Emphasis mine] – Fillmore W. Galaty, Wellington J. Allaway, & Robert C. Kyle, Modern Real Estate Practice, 14th ed., p. 16

This is precisely why, in the U.S., it is possible for city councilmen to collect a portion of land rent through property tax levies, yet be lawfully excluded from the land itself by whoever holds title to that land. Although the local government in this case has a legal right to a certain percentage of the land’s rental value, the titleholder has all the other rights of the aforementioned “bundle.”

Not only would the titleholder retain those rights under a geolibertarian system, those rights would be strengthened by the fact that (1) he would no longer be taxed for being productive, thus making it far easier for him to afford whatever the rental charge is, and (2) the law would require any surplus revenue to be distributed equally as a citizens dividend. (The latter would provide a built-in incentive for citizens to bring enormous pressure to bear on government to limit its spending, since less wasteful spending would mean a greater surplus, and thus a higher dividend.)

6.3. What is the difference between ownership and possession?

[I haven’t finished writing this section yet. It takes time to write stuff.]

For now, see: But private land ownership is necessary to resolve tragedies of the commons.

6.4. What do Georgists think about the Labor Theory of Value (LTV)?

The Labor Theory of Value is circular and vacuous. If it was just “The value of X is the amount of labor necessary to produce X”, then it would just be stupid, but not circular. But the LTV is defined as “the value of X is the amount of socially necessary/useful labor required to produce X”, and “socially necessary/useful” presupposes value.

A better alternative to the Labor Theory of Value is Marginalism. u/greenmeklar has written some great essays on the problems with the LTV, and why Marginalism is a better economic theory.

6.5. What do Georgists think about equality?

One of the goals of Georgism is to reduce wealth equality, not to eliminate it. If we made everyone 100% equal in wealth, then other inequalities would matter more anyway, e.g. genetic inequality, sexual inequality, moral inequality, etc, so it’s futile to make absolute wealth equality an imperative.

We believe in equal opportunity, not equality of outcomes. The latter would reduce the production of wealth, make everyone worse off, and cause tyranny. Many people compare Georgism to Communism, but they are very different:

  • Georgism is motivated by a positive vision for the future, whereas Communism and Socialism are motivated by hatred of those who are wealthy and successful, and they fantasize about killing billionaires, landowners, and/or politicians.
  • Georgism makes a distinction between ownership and possession. Georgists are okay with the wealthy and the elite maintaining possession over their land (and ownership over the fruits of their labor), as long as they compensate everyone else for being able to have that right. By comparison, Communists and Socialists want to steal property from the wealthy and redistribute it for everyone else to own.
  • Georgism is supported by solid classical economics. Communism and Socialism are not.
  • Georgism has never been tried before. Communism and Socialism have been tried, and they failed.
  • Georgism doesn’t enable rent-seeking, nor does it cause free-rider problems, nor does it penalize people for being productive. Communism and Socialism do all of these.
  • Georgism would reduce corruption, compared to alternative proposals for bootstrapping market prices. If anything, implicit ad hoc pricing creates even more opportunities for corruption and evasion, in comparison to taxing natural resources at the point of extraction.

I think it’s theoretically possible for Georgism to be enforced by non-elites if the power structure is just right after great reform or a major revolution, but I also wouldn’t be surprised at all if the world’s natural resources ended up being controlled by the elites, since that’s pretty much the same outcome of every ideology (that the elites take over everything). Even if Georgism resulted in the elites seizing power to manage the pricing of land and natural resources (and thus the distribution to some extent), I would still view it as preferable. Even the elites have an interest in conserving natural resources for a sustainable future, which natural resource taxation would achieve, along with all the other benefits on this list.

6.6. What do Georgists think about landlords?

[I haven’t finished writing this section yet. It takes time to write stuff.]

6.7. How would Georgism reduce wealth inequality?

When everybody owns land equally, this ensures that everybody has equal access to the three factors of production: Land, Labor, and Capital. It follows that economic productivity is increased, and wealth inequality is dramatically reduced. The massive wealth inequality that exists today is in large part caused by the skewed and unequal ownership of highly valuable land.

Since the wealthiest people are naturally the ones who own the most land and the most valuable land, Land Value Tax would therefore reduce economic inequality, because the LVT would mostly fall on the wealthiest members of society.

Additionally, Georgism would make housing more affordable. When housing is unaffordable, it tends to affect the lower classes more than the upper classes, so that’s another way how Georgism will reduce wealth inequality.

As a thought experiment for illustrating how land is the ultimate driver of economic inequality, imagine if there were no scarcity of land. Imagine so much high-quality, easily accessible land that anyone can use as much as they want without reducing the amount available for others to use. How many of the ’problems of capitalism’ would still remain in such a world?

6.8. Wouldn’t the wealthy simply find some way of evading land value taxes? -N

You can’t move a nation’s natural resources off shore. You can’t evade a Resource Rentals charge, just as you can’t evade paying your rates or mortgage, whether you’re Joe Bloh or James Packer. In our current system if you don’t pay your rates or your mortgage your site is reclaimed by the owner or by the local authority. That wouldn’t, and shouldn’t change. If you want to own shares or property overseas that should be no skin off anyone’s nose, good luck to you. If you’re using wealth generated in this country to do it, that’s fine too because you would also be spending some of the profits here anyway.

6.9. Wouldn’t the wealthy sell up and flee with their wealth? -N

Few would flee; they would stick around and utilise their wealth in productive enterprises which would allow them to utilise their managerial or entrepreneurial skills to generate more wealth for themselves and, through resource rentals, for the community at large. People could shift cash, gold jewelry, antiques, paintings or any other moveable assets offshore to their heart’s content. They could use all the modern electronic means at their disposal to move funds, and do whatever they like with their money. Anyone who’s willing to work for a living rather than be supported by the labor of others will be grateful. If there is a flight of non-productive capital, the only function of which was to hold land and resources out of use in expectation of some future unearned profit, then their departure would not be missed in the long run. As these non-producers fought to get rid of their unused sites, the prices would drop, and Mr. and Mrs. Joe Bloh would be able to realise the dream of owning their own house. Joe and his partner might also be able to start up a little business and produce for themselves a passably good life. In fact the whole population would have a much better chance of enjoying the fruits of their own labor.

6.10. Wouldn’t LVT discourage people from owning land?

LVT does discourage people from owning land. That’s a feature, not a bug. It ensures that people only use as much land as is efficient for them, leaving the rest for someone else to use. That’s what we want, as opposed to the land-hoarding and rentseeking behavior that dominates our current real estate market. (Actually, we want to raise the tax so high that the sale price of the land becomes zero, erasing the notion of privately ’owning’ land at all. Everyone would be a tenant on common land, paying back everyone else for the land they use and, conversely, getting paid for all the land they don’t use.)

6.11. Wouldn’t the LVT make it more difficult to own land, especially for poor people? -N

No, because land rent, as mentioned before, gets paid either way – regardless of whether or not it gets diverted into the public treasury.

Even when you pay the sale price of land, you are paying land rent, since the sale price is simply the rental value divided by the interest rate. And since land is in fixed supply, decreases in land value taxation are invariably capitalized by titleholders into higher rents and land prices. Thus, people in general, and the working poor in particular, end up paying back in higher rents and land prices what they presumably get from the tax cut; and pay back even more in terms of (1) a lower margin of production (and thus lower pre-tax wages), and (2) a heavier reliance on wage and sales taxes.

So once again, it is not a question of if land rent gets paid, but to whom and on what basis – to a fraction of the population, on the basis of the earth being “owned” by a relative few; or to everyone equally, on the basis of the earth being that to which all have an equal right of access? Geolibertarians believe it should be the latter, since that is the only just and practical way of establishing true equality of opportunity without enforcing equality of outcome in the process.

As for poor people, the LVT would actually make it much easier for them to acquire land, since it would reduce the artificially high price of land, as well as increase wages by raising the margin of production, on the one hand, and reducing the need for wage taxes, on the other.

6.12. Why would anyone want to own land under this system? -N

At present, wages gain about 3% p.a, the sharemarket 7-10% and now land, instead of the 15% plus gains of recent (on a huge lump sum amount), would be left with about 10% of a much smaller increased land value ie a 10% share of whatever gain on a $45,000 Site Rental versus the old 15% on $450,000. Owning land would thus still see competitive returns to what can be earnt in banks or the sharemarket, just not as large a capital gain as the lump sum would be greatly reduced in size and growth. For more see Land Price versus Land Value.If the person is a renovator, they are entitled to all of the gains from the improved building.

Also, with the speculative component removed, the land market would be less volatile, reducing the risk of land ownership. Cheaper land values would give our youth hope.

6.13. How can homeowners pay a land value tax, if their land goes up in value and their wages do not rise? -N

If the majority of land rents were collected by the community, the land market would become much less volatile. Speculative booms and crashes in land values would be greatly dampened, if not completely eliminated. During a transition period, in which land values were becoming a greater share of public revenue, there would be an opportunity for companies to provide insurance against an unexpected increase in the land value tax. The insurance would have a cost at the time of purchase, so that the new titleholder would know if he could afford the payments. Also, retired folks with low incomes could postpone the payments until the property is sold or inherited.

6.14. A rich man has a large mansion; a poor widow has a small house on an adjoining lot with the same value. Is it right that they both pay the same tax? -N

There is no reason in justice why the community should not charge poor widows as much for monopolizing valuable land as it charges rich men. In either case it is a special privilege which should be paid for. In our sympathy for the rare widower in this situation, let us not forget the hosts of working people who not only do not live next to mansions but have no place to live but by some landlord’s consent. They would find it much easier to get a place to live under the Single Tax than now.

7. How Georgism Would Affect Different Interest Groups -N

7.1. How would Georgism affect wages?

[I haven’t finished writing this section yet. It takes time to write stuff.]

7.2. Wouldn’t taxing land at 100% of its value be unfair to landowners?

It would be if 100% LVT started getting applied overnight, but only because many people’s wealth and life savings are stored in the value of their land, especially for many middle-class homeowners.

Any conversion over to a Georgist taxation system would have to be a very gradual process, taking at least 30 years in order to give everybody enough time to re-adjust their personal finances, especially for the people who are relying on land speculation as part of their retirement portfolio. But once society is through that, the economy will be better off than it was before the transition and it will be smooth cruising from there on out.

For as labor cannot produce without the use of land, the denial of the equal right to the use of land is necessarily the denial of the right of labor to its own produce. – Henry George, Progress and Poverty, Book VII, Chapter I

7.2.1. If someone buys land in good faith, under the laws by which we live, would that person not be entitled to compensation for individual loss if we taxed away the value of his land? -N

Even at present, if a landowner does not pay taxes, his or her land is confiscated by the government without compensation. Land grants and taxation are clearly matters of the general public policy; they are legislative and not contractual in character. Titles to land values and privileges of exemption from taxation are voidable at the pleasure of the people. The reserved right of the people to terminate grants of land value is a part of every grant of land.

Since Progress and Poverty was written, there has been a considerable body of public opinion in favor of land value taxation, and the proposal has been put into application in several parts of the world. Notice, therefore, has been served that there is an effort in progress to accomplish community collection of rent by proper methods. As this movement grows, people cannot be allowed to make bets that it will fail and then, when they lose their bets, to call upon the government to compensate them for their loss. Note too that land titles will remain. The land will be just as good as before — even better — for building or producing.

7.3. Wouldn’t Georgism give landowners more influence over the government since they would be paying all the taxes?

It’s not clear why non-taxpayers wouldn’t have a voice. They would probably have just as much representation as they do now under this current system, and even if that’s not very much, there are ways to improve the structure of the government so that it listens to all its citizens, but that is very complicated and a discussion for another thread.

Keep in mind that we are also in favor of head taxes since we believe that everybody who benefits from government services should be required to make a contribution for receiving them. Head taxes don’t reduce economic productivity since they are a flat rate that is applied to every citizen.

7.4. How would Georgism curb unemployment? -N

Under this proposed reform, land and resources would no longer be kidnapped and held out of productive use by private monopolies. Access would be within the reach of anyone who was willing to reimburse the community for the community-created value of the site. Wage levels, uninhibited by fear of unemployment, would flourish as demand for labor surged. People given a free hand to produce and to prosper, and knowing that they would get as much income as they wanted to work for – or as little as they needed to comfortably survive – would begin to do the sorts of work that pleased them. The quality and range of goods and services would increase dramatically, thereby improving their attractiveness to our trading partners.

7.5. Wouldn’t land value tax hurt farmers? -N

Nope. Georgism would help everybody, including farmers.

Land Value Tax is not the same thing as Land Area Tax. Rural land tends to be worth far less per unit area than urban land, so it’s not as big of a deal as you might think. For instance, in the United States, urban land constitutes less than 3% of the total land area but over 70% of the total rent (and thus potential LVT revenue). So farm land could only account for less than 30% of the total, which is a very small portion of the land rent, especially in proportion to its size. If anything, farmers might receive more money via the Citizen’s Dividend than whatever that they might pay in LVT.

For another thing, LVT would: 1. densify suburbs that sprawl into farmland, and 2. tax land speculators. Both of these factors would only make rural land cheaper for farmers.

No, it would help farmers. In the first place, the LVT would fall primarily on urban land, not rural land, since land values are concentrated primarily in urban areas. In the second place, the increased cost of paying a higher tax on land value would be more than offset by (1) the savings incurred from paying lower taxes on everything else, (2) the reversal of urban sprawl (and thus of the inflationary pressure that sprawl currently imposes on the value of farmland), and (3) the increase in income that would result from both a higher margin of production and a surge in overall economic activity. For supportive empirical evidence, see the following:

The critical point is that the Site Rent is levied on unimproved (no structures, fences or other improvements) values. In marginal country this would be very little indeed. In high rainfall fertile country it would be more since the unimproved land itself is more productive. However, it would still be minuscule compared to current mortgage levels. It is ultimately the market which decides the value anyway. Once you’ve paid the site rental, you keep what your earn, and pay no tax, direct or indirect, hidden or not, on anything else. You keep the fruits of your labor, and you earn more or less as much or as little as your desires demand. Productive farms pay less under a proper Site Rental system than under CIV (Capital Improved Value), as they generally have developed their farms with more buildings and machinery. Farmers lobbying for CIV have generally been hoodwinked by the real estate lobby, who prefer CIV as then households and business subsidise vacant lots.

Here’s a more extensive answer to this question.

7.6. How will Georgism affect multinationals (MNC’s)? -N

If Community Site Rent were introduced, then, wages would rise, and the following would result:

MNC’s and large businesses typically rely on large sites with big carparks. As large land users they will have to pay higher site rents than your average small hardware store, helping to restore the balance, moving us towards re-localisation.

Consumers, having more disposable income, would not be forced any longer to opt for the cheapest product, and would be in a position to take ethical decisions on what they do and do not buy.

The result of this would be that any MNC’s which were using unethical practices would lose sales against more ethical competitors

If MNC’s had production sites in this country they would have to offer higher wages to hold their workforce, which they could may struggle to do whilst remaining competitive. This may lead them to close down operations here and move the production offshore to a cheaper labor source.

If the MNC derived a significant part of their income from ground rents, either directly or indirectly, (i.e. through cheaper labor or raw materials), then they would probably be forced to cease operations in this country anyway if they wished to maintain the same levels of profitability for their international shareholders.

7.7. How will Georgism affect lenders and borrowers? -N

The vast bulk of unproductive sites, including those held for speculative gain or in anticipation of upzoning, would come onto the market. Unimproved land values therefore would initially fall which would in turn make them more accessible to all, whether first home buyers or commercial enterprises. The site rent, being based initially on a percentage value therefore would probably produce less revenue from existing occupied or utilised sites, but since there would be a large increase in the number of utilised sites, the total revenue would remain or more likely increase.

The cost of mortgages would drop because the land component would cost less – and in a very short space of time, it’s sale price would drop to zero and all you would be paying would be its annual rental value. In this scenario, you would only be borrowing to the value of a year’s site rent at the most for the land component, after which time you would more than likely earned enough to easily afford the next years rental. You would still be paying off the money borrowed on the building. As well as this, because everyone would retain a much larger portion of their earnings, they will be able to come up with a larger deposit, and also to pay off any loan much more rapidly. The demand for money will drop, resulting in interest rate falls.

7.8. How will Georgism effect new and long-term mortgagors/owners? -N

Those who have, not long prior to the changeover, taken on a mortgage over a residential or commercial property may feel ill-served by such a site rent for revenue system as they will be obliged to pay both the mortgage repayments, and at the same time be liable for the community site rent. They are no worse off though and in fact are probably better off, because;

Employment conditions & wages will greatly improve due to the increase in small business. Other spin offs include a safer society, better public transport and reduced overall household debt pressures. The long-standing mortgage payer has been liable for a comparatively similar amount, but over a longer period, but will never the less still be liable for the same site rent. For the newer mortgagor, in all likelihood interest rates in the new system would decline dramatically, due to both a decline in site values and therefore demand for finance, making the mortgage effectively cheaper. On top of this, both the above mortgagees will from the date of changeover be able to retain a much larger portion of their income, since all taxes will have been abolished.

7.9. How will Georgism meet the needs/demands of Entrepreneurs and Manufacturers? -N

Opportunities would abound. Large corporations which now rely on making maximum profit from cheap, shonky and second rate goods would face serious competition from smaller operators with lower overheads (due to site rents), so would have to improve their game or disappear. People would have much more money to spend, and would therefore create many more new markets for the enterprising entrepreneur. It should be remembered that if, prior to the change, the entrepreneur was getting the bulk of their income from productive work, not from site rents, then they would be bound to gain from the proposed change. Those with capital already at their disposal could if they wished put it to use establishing new enterprises or improving existing ones, and so benefit from the new system which allows them to retain a much larger portion of the wealth they generate, while at the same time returning to the community its rightful share of the community generated site value.

7.10. How will Georgism meet the needs/demands of Exporters and Importers? -N

All tariffs and duties would disappear. Local product would be cheaper to produce, of better quality, broader ranging, and more abundant than ever before. Our products would fetch a premium on the world markets. Imports would be cheaper. We could economically import the very best capital equipment and use it to produce even better products for home consumption and re-export. Consumers would have ready access to the best the world had to offer in the way of manufactured goods and other products

7.11. How will Georgism meet the needs/demands of Environmentalists? -N

Pollution, toxic wastes, erosion, salination, land degradation, over extraction, decimation of tropical forests, diminishing fish stocks, threatened wild life; Private monopolisation of land and resources is the direct cause of all these environmental problems. Leases and royalties charged to the extractive industries do not reflect the true value of the minerals and other resources extracted. The true cost of these items must include the cost of ensuring the absolute control and minimisation of any resultant environmental degradation. If this resulted in the prices of some of these resources increasing, then that is what we must be prepared to pay. In the manufacturing industries it is cut-throat competition among producers which leads to much of the current pollution and environmental degradation, and tempts those involved to cheat and to side-step regulations etc. in order to survive. With a site rent for revenue system they could afford to comply with the strictest environmental controls.

7.12. Who wins; who loses? -N

Broadly, anyone who gains part of their income from site rent will lose that part of their income through the Community Site Rent . Whatever part of their income derives from their own earnings will not be taxed any longer and will remain entirely in their hands. They will keep what they earn and no more.

The biggest losers therefore will be those who rely entirely for their income from site rents, and who do not earn their living by contributing to the production of wealth. The further down the scale one goes in the ratio of income from site rents compared to that from productive work, the better off the individual will be. At the very lowest end of this scale, where the individuals income is made up entirely of what they earn, then the maximum benefits of the system will apply. In other words the people at the bottom lose least, those at the top lose most. You could not have a more progressive revenue system than that! Thus we have a total inversion of the age old pattern of the community’s wealth aggregating towards the top at the expense of all at the bottom.

It should be remembered that those with large fortunes at the time of changeover would not necessarily be disadvantaged. Certainly they would lose whatever part of their incomes which were derived from the monopolising of community resources, but they would still retain all their existing capital, which they would be free to invest in productive, wealth-generating enterprises, and could utilise their managerial and entrepreneurial skills. They would not lose their yachts, island hide-aways, mansions and penthouses, rural acres. Nor would they need their battalions of accountants and tax minimisation experts.

7.12.1. Though some people have made money by owning land, haven’t others lost? Do not the losses offset the gains? -N

Possibly; but what the land speculator loses, the community does not gain. What the land speculator gains, however, the community does lose. As between land speculation and the community, losses cannot be justly charged against gains. The taxation of land values, incidentally, will put an end to these “unearned losses” as well as to unearned gains.

7.13. Won’t it fall harder on the prince in his mansion than on the pauper in his hovel? -N

Don’t forget that the proposed Community Site Rent is struck initially on unimproved values. So the value of the building, be it hovel or mansion, is irrelevant. But to answer the point regarding equity;

  1. The owner of a site of unimproved value $1m will pay $100,000 nominal site rent in the first year, and in subsequent years whatever the annual assessed rental value is – in fact probably around 10% of the site’s notional capital value, even though “capital value” or “price” will no longer exist.
  2. The owner of a farm, whose 2000 hectares, excluding any improvements is worth $100,000 will pay $10,000 nominal site rent in the first year, and in subsequent years whatever the annual assessed rental value is. This farmer will pay no tax on his income from the crops, no tax on the farm machinery, new car, furniture, fencing materials, food, clothes, beer, books, new computer, TV set, home entertainment centre, RM Williams boots.
  3. The owner/occupant of the fibro hovel would be unlikely to be living next door to the mansion, and more likely be in an outer suburb sitting on land worth maybe $40,000 at current values. He/she will pay $4000 nominal site rent in the first year, and in subsequent years whatever the annual assessed rental value is.
  4. But what would all these people be paying now in direct and indirect tax? The millionaire; probably 5 or 10% of his/her true income in income tax, but the same indirect taxes as everyone else. The average wage or salary earner, between 30 and 50% of their income on income tax and then the same as the millionaire in indirect taxes on everything else. A minimum of 50% of average incomes disappear in direct and indirect taxes, an increasing proportion of which goes to support an ever more unwieldy and expensive administration. You don’t start earning a clear wage until more than half way through the year.

7.14. How would old age pensioners on valuable sites be able to pay the site rent as suggested? -N

Pensioners would be better off, but will be faced with some difficult questions in the beginning. At present day inflated land prices, the $330,000 house would see the unimproved price at $230,000. Ten percent of this is $23,000 in site rental. This would be a shock to most retirees. However, it must be remembered that we are presently at record highs for housing, and a site rental would return property prices back to more realistic levels within a few years. Thus in a year’s time 30% of the speculative price has probably been brought out of the price. Site rent then falls to $14,000. It should fall back again to about $10,000. It is likely that a sole pensioner on $15,000 will initially not be able to cover their site rental. They should sell and move to a smaller premises. Alternatively, they could do a reverse mortgage and pay their site rental from their estate. That is a negative but look at the positives:

  • their grandkids can now afford a house, reducing future generation’s reliance on any inheritance
  • they can walk the streets in comfort as crime drops due to increasing small business, employment opportunities and wages.
  • their pension’s purchasing power will be much higher, giving them more dignity, as the removal of direct and indirect taxes will boost pensions by 40%. These are just some of the gains, making this small sacrifice well worth any short term transitional issues.

A couple on a pension will be able to continue as per normal, though they may have to tighten their belts for the first year. The added purchasing power of 40% will more than make up for the short term pain of the transition.

Since governments would be in surplus they would happily, with no pressure from the community, increase pensions to a level which provided a comfortable retirement to those who had through their working life contributed by their very presence to the wealth of the nation.

It’s worth bearing in mind that this reform is not simply another slightly more novel form of tax. This proposed Community Site Rent recaptures and returns to the community at large what was previously being diverted into private hands. It allows what was previously paid out, with considerable pain, through a complex and massively inefficient array of taxes to remain in the hands of the people who produced it. For this reason alone, and for the justice of the principle on which it stands, the Community Site Rent demands serious consideration.

It the worst came to the worst, and the occupier was too ill or incapable, then the site rent debt could simply be deferred until the death of the occupier, and then retrieved from the estate, as are mortgagee debts and rate arrears now.

After spending a lifetime seeing some people work blood, sweat and tears for little return and others greasing the system through property speculation, many retirees would be able to look back in satisfaction as being the ones who made a small sacrifice for the greater good.

7.15. What happens to the self-funded or partly self-funded retiree? -N

These deserving retirees would be overjoyed with such a scheme. Although they would indeed have to continue repaying the community for the value which the existence of the community itself gives to the site, this would be truly small beer compared to what they’re currently being stung. When you consider that on top of their income and capital gains taxes, they are paying up to 32% of every dollar on a truly biblical multitude of taxes including income tax for the producers and providers of goods and services plus all the other indirect and hidden taxes we all endure.

If the unimproved value of the site that these retirees occupied was worth say $100,000 (excluding the value of the house), then they would be paying a maximum $10,000 nominal site rent in the first year, and in subsequent years whatever the annual assessed rental value is. The buying power of what’s left grows by at least a third since they’re paying no income tax or capital gains tax, nor are they paying any of the other multitude of hidden taxes and imposts. They’d be rapt.

7.16. Will pensioners and retirees – currently untaxed but living in their own property, be liable for site rents as well? Will they be reimbursed for past taxes paid? -N

They will pay site rent and they wont be reimbursed for past taxes. We must look forward. We hope they will see the benefits to society, in particular their grandchildren, worth the change in thinking. However, as we have just seen (above), pensioners will gain plenty from this new perspective. They are already being taxed even if they are paying no income tax. They are still paying rates, possibly a mortgage or rent, possibly for the slightly better-off ones, capital gains tax. Crucially, the many indirect taxes and hidden imposts hit the hip pocket to the tune of about 40 cents in the dollar. Now if all of these charges are removed, they would come out and march for this scheme if the current taxation system only left them a few spare cents for the bus fare to the march assembly point.

7.17. Unemployed adults – currently untaxed but perhaps owning their own home? Will they have to pay as well? -N

Yes, but again, as in the case of the pensioners mentioned above, they would be markedly better off. Bear in mind too that such a scheme would very quickly eradicate involuntary unemployment, as there would be a large increase of buying power in the community at large which would naturally lead to a demand for workers. Small business would boom, giving the unemployed other options outside of telemarketing.

7.18. New enterprises – yet to realise any profit but paying tax on the property from which they operate? If you tax them, won’t they fail? -N

Don’t forget, all other taxes and imposts would be abolished. No provisional tax, no taxes at all! It would be a golden day for anyone wanting to set up a business, or expand an existing one. As things stand now for small business, and for bigger business for that matter, not only do they have to cover such things as payroll taxes, all of the import taxes and other subtle add-ons, sales taxes and other rubbish, but if their business does look like its beginning to finally make a profit, lo and behold, the landlords not only up the rent, buts stick their hooks in for a percentage of the turnover to boot. Many more businesses under this proposed system could afford to be their own landlord. Any entrepreneur worth his sea-salt would join those old pensioners on the march. In fact he or she might even subsidise their bus fare.

8. Urban Planning Questions

8.1. How would Georgism affect urban planning?

[I haven’t finished writing this section yet. It takes time to write stuff.]

Cars just aren’t sustainable in general. Even with fracking, the world will run out of oil eventually at some point. And since EVs aren’t sustainable either, the world will have no choice but to give up on cars within the next few decades or so. The best sustainable solution that the West can hope for is high speed rail and Georgism.

Also see: The Immense Social/Political Power of Car Dealerships - Polymatter

8.2. How does Georgism reduce urban sprawl?

[I haven’t finished writing this section yet. It takes time to write stuff.]

i

8.3. Wouldn’t we run out of space? -N

No, we wouldn’t necessarily be occupying more space, we may even be occupying less but doing it much more efficiently, productively and not environmentally damaging or resource indulgent (i.e. requiring road networks and separate infrastructures out to geographically isolated satellite suburbs.)

Lack of land supply is the property sector’s latest diversion plan to real analysis of housing un-affordability. take a cycle around your community and you will soon see a large number of vacant blocks of land and empty houses. This is where the real land supply problem is. We don’t need any more public land supply (and endless urban sprawl), we need this private land supply, currently locked up to enhance capital gains, encouraged onto the market. Site Rental is the most efficient way to do this.

8.4. Why not just build outwards instead of upwards?

It’s true that cities could build outwards instead of upwards if there is still uninhabited land available and the geography allows it. However, it still remains the case that the supply of valuable locations within the jurisdiction is fixed. Not only that, but building outwards would increase urban sprawl and it would require significantly more infrastructure to maintain, compared to if cities built upwards instead. It’s highly unlikely that the Earth would have the resources to support Humanity if cities kept building outwards instead of upwards.

While there are millions of square kilometers of uninhabited land around the world, asserting that statement ignores that not all of that land is equally valuable.

The problem is that if land is not taxed, there is no guarantee that it will be used as efficiently as possible. There’s a huge difference between building a suburban style house in Manhattan versus building an apartment complex in the same location.

8.5. If central land in a city is high in value and low on the fringes, doesn’t LVT just encourage cities to sprawl and result in poor land use?

The reasoning behind this question assumes that the LVT only imposes negative consequences, it ignores how LVT would discourage land speculation and other inefficient land usage, and it ignores how the highest LVT parcels have high land value for a reason: because everybody is willing to pay higher auction/assessed values to possess those parcels for themselves.

8.6. If you tax land value, then won’t all the houses be crammed together and poorly built in order to use as little land as possible?

Nope, that doesn’t follow at all. LVT incentivizes building more housing. When you increase the supply of housing, you increase competition inside the market. Competition decreases prices and increases the quality of the goods and services available to the consumers.

It is also wrong to predict that all the houses and apartments would be crammed together. If anything, that’s what you see or will see in the current system. Housing is so freaking expensive in Japan that there’s literally coffin-sized apartments and POD hotels that people live inside of since they have no other choice. And Western countries are bound to go down that road too in some areas if they don’t enact housing reform. It’s already happening in some US cities.

Obviously, the vast majority of people don’t want to live inside coffin-sized houses, so they’d absolutely be okay with paying more LVT if that buys them more living space. And since landlords would have every incentive to get as many customers as possible, they would have to build something that people want to live in.

We can make an analogy involving the sizes of various air flight seats. Even though the smaller economy class airline seats are obviously cheaper, plenty of people don’t settle for them. Plenty of people are willing to pay more money for more roomier seats on their flights, because they believe the extra room is worth the extra cost. Similar spatial dynamics apply to housing.

8.7. How would eminent domain work in a Georgist Society?

LVT would cause Eminent Domain to be less one-sided since more valuable properties would be more consenting to the eminent domain, since they would have to pay higher LVTs.

If the city collects a LVT and can also decide which land will get purposed for different things, it can essentially operate as a business, which should make it and all its operations run more efficiently.

8.8. How would Georgism prevent gentrification?

[I haven’t finished writing this section yet. It takes time to write stuff.]

9. How would Natural Resource Taxes be calculated?

There are tech startups like Value Base, which use machine learning algorithms to calculate land values.

9.1. Can land value be seasonal?

Yes.

[I haven’t finished writing this section yet. It takes time to write stuff.]

9.2. Would natural resource taxes have a fixed rate?

We want to capture 100% of the land rent, whatever that is, and even if it changes over time.

It doesn’t correspond to a ’rate’ on the land’s sale price as in a traditional property tax. The sale price itself is sensitive to taxation (you’re not willing to pay as much to buy something you have to pay more tax on), and capturing 100% of the land rent would effectively drive the sale price to zero, making the traditional calculation useless. Therefore we need other methods of calculating what the land rent is, that don’t depend on sales. Fortunately that is relatively straightforward and the statistical tools largely already exist.

[I haven’t finished writing this section yet. It takes time to write stuff.]

9.3. Why should land be taxed at 100% of its value? Why not a different percentage?

[I haven’t finished writing this section yet. It takes time to write stuff.]

9.4. How much land tax value would be “fair”?

[I haven’t finished writing this section yet. It takes time to write stuff.]

(Arguments which I have presented to actual marxists and for which they of course have no coherent response.)

9.5. Would all land value have to be appraised?

Land only has value if people want it. There’s not much sense in putting a price on the land in Antarctica or the Sahara Desert because no one wants it.

But yes, all land that people want would have to be appraised, in order to know how valuable land is compared to other land. The two main proposals are either by land value appraisal or auctions.

9.6. What is meant by ’unimproved’ value? -N

The “unimproved value” of a site is simply its worth discounting any improvements – the land value itself. It is the current market price less the current value of all improvements. As happens now, it is the market which determines the value of a site and anything on it. The desirability of a site for residential, commercial or industrial development, for the establishment of a mine, or for exploitation as a fishing-ground, is what creates its value. A derelict site fronting a shopping mall will be worth by and large what its neighbouring sites are worth. This fact is demonstrated when a vacant site goes to public auction. The amount of the successful bid determines the value of the site. The unimproved value of farmland in a marginal rainfall area is likewise determined by public bidding.

Any work carried out on a site, be it a multi-story exclusive store or a wheat crop, are improvements. The full value of these improvements should rightfully belong to whoever produced them. Our present system of local Council rates incorrectly rates on improvements, under the Capital Improved Value system (CIV). This penalises landowners for developing and improving their property. It also means that households pay a higher share of the tax burden than the wealthy property speculator who owns vacant land. No wonder our suburbs are sprawling for miles.

9.7. How would the value of improvements to the land be separated from the land value itself?

For starters, it is already common practice in the real estate industry to separately evaluate of land values from property values. If you can determine the value of the structure on the land, then the value of the land is essentially the difference. So this is pretty much a non-issue.

“Land value represents the present market value of the land. It does not include the value of improvements. Land value is arrived at through an analysis of current sales of comparable land in the general area. It is computed separately because land is not depreciable.” [Emphasis original] – William L. Ventolo, Jr., Ralph Tamper and Wellington J. Allaway, Mastering Real Estate Mathematics, p. 115. The only people who seem intent on ignoring this fact are opponents of the LVT.

Furthermore, if one argues that it will never be fair nor possible to separate the value of improvements to the land from the value of the land itself, then they would also have to reject the validity of many other concepts.

[I haven’t finished writing this section yet. It takes time to write stuff.]

10. Revenue Questions

10.1. Site Rent for Revenue is a crackpot idea. -N

We would be happy to acknowledge this claim if you could logically and rationally refute the basic premises behind it, which are that; • Before the community arrived there was no intrinsic value in land or resources. • The advent of communities created demand and competition for sites which then gave them value. • Since it is the presence of the community itself which created the value, then this value must by right belong to the community and not to an individual or group which throws up a fence around it. • This community created value therefore would seem to be the logical and by far the most equitable source of revenue for its government, and the community therefore has not merely a right but a duty, through its elected representatives to reclaim this community created wealth as the only fair and just source of public revenue. • This fund would be adequate to enable the government to carry out all of its proper functions in the service of the community, and so would enable them to abolish all other taxes, tariffs and imposts. For evidence, please see this Ground Breaking work by Tony O’Brien “Why put the entire burden of taxation on one area of economic activity? This distorts people’s spending decisions and reduces GDP. It’s just as foolish to do this by taxing only the use of land as it would be to tax only income generated from labor!”

Answer: Income derived from holding land or resources out of use is not productive activity. Bear in mind that this proposed site rent is on unimproved values. If you owned a site and put up a house, shopping centre or industrial complex, then you have every right to the rent or sale prices which those improvements generated, but you have no right to take the increase in site value which resulted from the very presence of the community and the infrastructure which was built with their wealth. It is incorrect to equate land with labor. Land is a finite resource in whose production we had no hand. Labor and its resourcefulness and potential is limitless. The adoption of this system, far from reducing GDP, would see it skyrocket.

10.2. Are natural resource values sufficient for generating the revenue needed for funding the government? -N

The answer to this question depends on (1) how you interpret national income figures, (2) what you consider to be the “legitimate” functions of government, (3) the extent to which a reduction in taxes on labor and capital would drive up the rental value of land (and thus revenue capacity), and (4) the extent to which shifting to a land-based tax system would increase economic output (and thus the tax base). With respect to national income figures, many economists accept (seemingly without question) the Commerce Department’s claim that land rent makes up only 2% of the national income. Assuming for the sake of argument that this is true, that means, with the national income at roughly $10.8 trillion as of last year (2005), a land-based tax system could yield little more than $216 billion in annual revenue. Not all economists, however, subscribe to the belief that rent constitutes only 2% of the national income. For instance, in The Losses of Nations (1998), Fred Harrison explains how a study by Wall Street economist Michael Hudson revealed that the revenue capacity of land is about 14% of the national income, or what in 2005 would amount to approximately $1.5 trillion in annual revenue.

With respect to the “legitimate” functions of government, there are some who consider all current expenditures (including corporate welfare and the insane drug war) to be “legitimate,” in which case the LVT would need to generate roughly $3.2 trillion in annual revenue for all levels of government. On the other hand, there are some who consider “legitimate” only those expenditures that go toward protecting individual rights (e.g. defending our national borders from military invasion, enforcing laws against force and fraud, adjudicating civil disputes, etc.), in which case the LVT would need to generate no more than $1.5 trillion in annual revenue for all levels of government.

With respect to the reduction of taxes on labor and capital, and the effect this has on the rental value of land, economists throughout history have observed that, when said taxes are lowered, land rent tends to rise proportionately. Why? For the simple and obvious reason that, the more people can afford to pay for access to a fixed quantity of land, the more titleholders tend to charge higher rents. If, for instance, the payroll tax were abolished, most of the resultant increase in take home pay would be absorbed by higher rents. Thus, it follows that the more the tax burden on labor and capital is reduced, the more the revenue capacity of land is raised by a comparable amount. (Economist Mason Gaffney explains this more thoroughly in Ch. 7 of The Losses of Nations.)

And finally, with respect to economic output, it is common knowledge that, all else being equal, an increase in output means an increase in tax revenue (regardless of the tax system in place). It is also common knowledge that, all else being equal, an increase in output means an increase in the rental value of land (regardless of whether land rent is collected publicly or privately). The question thus arises: to what extent would a land-based tax system increase output, and hence the tax base? On page 147 of “The Losses of Nations,” economist Nicolaus Tideman estimates that “…a shift to public collection of rent as the principal source of public revenue in the U.S. in 1993 would have increased the output of the U.S. economy by $1,602 billion above its actual level for 1993, implying that the U.S. economy is producing only 77 percent of what it could produce with a better tax policy.” [Emphasis mine]

All that being said, if you take the Commerce Department at its word on rent being only 2% of the national income; if you believe that current tax revenue outlays at all levels of government should be maintained; and if you ignore the extent to which both land values and economic output would skyrocket in the absence of taxes on labor and capital, then you will undoubtedly conclude that land values are not an adequate source of public revenue.

If, on the other hand, you agree with Dr. Hudson’s conclusion that rent is approximately 14% of the national income (if not more), then even if you oppose a moderate reduction in overall spending; and even if you ignore the increase in land values and economic output that would accompany any significant decrease in the taxation of labor and capital, the LVT would still allow for the abolition of the federal income tax. But if you believe that $1.5 trillion could easily fund the legitimate functions of government, and if you realize the extent to which both land values and economic output would increase in the absence of taxes on labor and capital, then you will almost certainly conclude, as I have, that land values are amore than adequate source of revenue for all levels of government.

10.3. How much tax revenue would Georgism generate as a percentage of a country’s GDP?

Probably about 40% in typical developed countries. If you do the math regarding the cost of housing, the level of existing tax burdens, etc, that’s roughly what it comes to. Could be perhaps ±10% from country to country depending on their population, dominant industries and level of economic development.

In the 2012 federal budget, tax revenue was about 1.73 trillion USD. However, total land values were 54 trillion USD, which is about 31 times more than the total tax revenue. A land value tax of 3% would bring in almost 20% of all current revenue. This is just pure land value, as in what the land is worth, not including the improvements or buildings. If we were to combine it with other taxes, it would be an absolute tax boom.

[I haven’t finished writing this section yet. It takes time to write stuff.]

10.4. How would rent be apportioned? With only one tax, how would it be divided between local, state and federal governments? -N

One proposal is to collect all rent at the local level: the local governments would pass a percentage to the state governments, which in turn would pass a percentage on to the federal government. This is the method usually used between state and local governments when property taxes are levied by the state. It was also used by the federal government when it levied property taxes between 1789 and 1861, apportioned among the states according to population, as specified by the US Constitution.

Another proposal is for each level of government to impose the tax directly on respective types of land — municipalities on land within their areas; states on land within their areas outside the municipalities, such as farm land; and the federal government on such natural resources as oil fields, mines, forests and waterways. In practice, this method is also partially observed.

There has been increasing debate in recent years over the disposition of transboundary resources. Debate is going on over nations’ rights to claim minerals on the ocean floor. The control of rivers that flow through the territories of many nations is a deeply sensitive issue. The advent of global atmospheric pollution has forced people to see the earth’s atmosphere as a resource that is shared by all. Extending the concept of the public collection of land rent to the international sphere can provide a peaceful and progressive way of resolving intensifying conflicts over resource access and use. Treaties concerning common property in Antarctica, outer space, and the world’s oceans have taken the first, halting steps in this direction.

10.5. If site values fall, won’t revenue fall too? -N

Some locations will drop but others, particularly those in central locations, may infact rise due to the immense productivity gains from our more streamlined taxation system.

Whilst the land price would fall to zero, the site rental would takeover as a yearly lease-type arrangement. The actual site revenue base will broaden with a vast increase in the take-up and utilisation of sites. There would be an initial decline in asking prices, which, in a static economy, would result in less revenue, but the point is that under a site rental system, much of the land which is now held out of use for speculative gain would be taken up and put into productive use either as residential land or in commercial or agricultural ventures. This would increase the amount of wealth generated by an unfettered, wealthier workforce. Since the wealth generated by the community would be so much higher than at present, and would largely remain in the hands of those who produced it, the community’s reliance on government-provided welfare and other resources would be reduced markedly, with a resulting decline in government expenditure.

10.6. “This tax, like a debits tax, ignores two of the basic principles of raising tax: -N

  1. A good tax should take into account the capacity to pay. This tax doesn’t — the unimproved value of land someone owns has no direct relationship to their income.
  2. A good tax should not distort saving and investment decisions. This tax does — only the land input into an economic activity is taxed, so people will substitute land for labor and capital. For instance it would encourage intensive farming of small areas of land, when it would actually be better to spread the labor and capital used more thinly over a larger area of land.”

On a):the classic cry of the ‘widower’ is a cunning tool by the real estate lobby to pull the heart strings on this entire issue. It is very effective. However, key central locations are very important to the community. Whilst change is not everyone’s cup of tea, anyone who has helped a lonely grandma with the garden or cleaning of her big old house will know that change can be as good as a holiday. The worst bit is thinking about it. Once the change is made to a smaller, more compact villa, the widow is re-invigorated and a new young family can move in and make good use of the property (rather than commuting 40mins plus to work as is becoming the norm for so many today).

On b)Site Rental is charged to encourage an optimal level, not a maximum level of output. The lower taxation burden and greater purchasing power on present incomes reduce the pressures to over-farm one’s resources. Yes, greater efficiency is encouraged in land usage, but rational decision making has room to breathe, such that spare paddocks are still financial to rejuvenate, giving them time to naturally regenerate for the next season’s livestock fodder. Balance is the key, not wastage. There is no such thing as a good tax, just as there is no such thing as an acceptable way of being robbed. Any revenue system which takes the bulk of your earnings in taxes, while allowing ‘in-the-know’ citizens to pocket the cream of community created wealth for themselves is simply condoning and abetting the constant and repeated committing of a crime. These are the tenets of a good revenue system:

  • It must be totally equitable. It must apply to all in accordance to the benefit received from the community. (with this system, a desirable block in Toorak might at a ten percent site rental, set you back $75,000 nominal site rent in the first year, and in subsequent years whatever the annual assessed rental value is. A block out the back of beyond for your caravan might cost you $20 nominal site rent in the first year, and in subsequent years whatever the annual assessed rental value is.)You pay in accordance to the benefit which the existence of the community gives it. Inbuilt in this is a natural ability to pay.
  • It must be easy and cheap to collect.
  • It must be unavoidable. Adam Smith, the godfather of economics, said a taxation system must reflect one’s ability to pay, have a certainty to it, be convenient and be efficient. Site and Resource Rents fulfill each of these criteria. No other method of taxation does.

It’s incorrect to suggest that income derived from holding land or resources out of use is not a productive activity. It is, since it produces income for the owner. Any income the speculator gets in excess of a fair wage to cover his efforts can not rightly belong to him. If he gets a windfall from the holding which he didn’t produce himself, then who produced it? The presence and activity of the community did, and it is from them that the speculator is taking the windfall and it is to them that it should be returned to in the form of a site rent. Any arrangement which does not do this is inherently unjust. Bear in mind this proposed site rent is on unimproved values. If you owned a site and bankrolled a house, shopping centre or industrial complex, then a return of a certain percentage from investments in productive activities is fine. After all the investor has provided risk capital to some enterprise which is going to produce some product for public consumption, and the returns to the investor simply reflect the level of risk and an inflation factor.

The investor in a site has taken a risk, and tied up their capital – who else could possibly be entitled to their capital gain? The investor is entitled to any gains made by the improvements they made to the site ie renovations, new buildings etc. However, the major gains we see in property prices are due to the actual increase in land prices, the unimproved value. Our taxes finance improved services. Our community development adds value to a neighbourhood. Baby-bonus type pressures contribute to demand for the limited places on the earth. So whilst the investor deserves about 10% of this capital gain, it doesn’t deserve the full 100% of the capital gain that the community as a whole contributes to. Our current system has masterfully managed a subtle system of subsidy for those already wealthy enough to own a piece of the planet.

11. Libertarian / Ancap Questions

11.1. Don’t seasteading technologies refute the argument that “land is finite”?

No, it doesn’t.

  1. This ignores the fact that we are losing more land due to rising sea levels than whatever land we could ever hope to gain with seasteading.
  2. Seasteading is extremely expensive since it costs billions of dollars, and it could never produce enough land to go around for everybody.
  3. There wouldn’t be a need to seastead more land in the first place if we used the land that we already have more efficiently, which we could most practically accomplish via LVT.
  4. Most land reclamation likely won’t be permanent. Even though Incheon International Airport was built on reclaimed land, it has been slowly sinking at an alarming rate every year. Climate change and rising sea levels will further worsen this problem. If the numbers are correct and this trend continues, then the airport is projected to sink below sea level within the next 100-200 years. This suggests that land reclamation won’t be a permanent solution to land scarcity.

11.1.1. Some people claim there are documented examples of land being produced. Doesn’t this refute the idea that land is in fixed supply? -N

No. Those who insist otherwise are confusing two different senses of the word land. In the every day sense, land usually refers to the dry surface of the earth; in the economic sense, however, it refers not just to the dry surface of the earth, but to the entire material universe, excluding humans and their products. In other words, land is not merely matter that occupies space; it is space. While matter can certainly be manipulated within that space, space itself cannot be added to or subtracted from. This is precisely why the value of “land” is often and more accurately described as the value of “location.”

“The essential feature of land is that its quantity is fixed and completely unresponsive to price.” – Paul A. Samuelson & William D. Nordhaus, Economics, 16th ed., p. 248

“Land has no production cost; it is a ‘free and nonreproducible gift of nature.’ The economy has only so much land, and that is that. Of course, within limits any parcel of land can be made more usable by clearing, drainage, and irrigation. But these are capital improvements and not changes in the amount of land itself.” [Emphasis mine] – Campbell R. McConnell & Stanley L. Brue, Economics, 14th ed., p. 604

“Land, which is the earth’s surface, is immobile. It is true that some of the substances of land are removable and topography can be changed, but still that portion of the earth’s surface always remains. The geographic location of any given parcel of land can never be changed. It is rigid and fixed.” – Wade E. Gaddy & Robert E. Hart, Real Estate Fundamentals, 4th ed., p. 9

“Remember: No one is making any more land.” – William H. Pivar, Real Estate Investing From A To Z, revised edition, p. 3

11.2. Doesn’t taxing people’s land violate the NAP?

No it doesn’t, and for several reasons:

  • Everybody has an equal negative right to land. The burden of proof falls on those who argue for unequal negative rights. Anti-Georgism violates the Libertarian NAP.
  • Anti-Georgism leads to an anti-free-market economy, and that certainly isn’t compatible with Libertarianism.
  • The NAP isn’t as “objective” as Libertarians would like to believe. This list shows a huge myriad of ways how Libertarians will disagree with each other, and it is evidence that Aggression is subjective, so the Libertarian Non-Aggression Principle is yet another subjective moral principle that does not form a basis for “objective morality”.
  • Private landownership violates the NAP. All privately owned land comes to be owned (originally) by claiming it, and then threatening or using force upon everyone who disagrees with one’s claim.

But the state would determine people’s right to land.

The state would not determine people’s right to land. The state’s duty is to enforce equal rights for all. That includes land rights.

11.3. Isn’t land-ownership the foundation of property rights, and thus of a free society? -N

No, self-ownership is. That is to say, the foundation of property rights (and the freedom that flows from those rights) is the property that each person has in himself and, by extension, in the fruits his labor. “Though the earth, and all inferior creatures be common to all men, yet every man has a property in his own person. This nobody has any right to but himself.” – John Locke, 2nd Treatise of Government, Ch. 5 “The property which every man has in his own labor, as it is the original foundation of all other property, so it is the most sacred and inviolable.” – Adam Smith, The Wealth of Nations, Bk 1, Ch. 10, Pt 2 “The property rights that each citizen has in himself are the foundation of a free society.” – James Bovard, Freedom In Chains, p. 86 “Libertarianism begins with self ownership.” – David Bergland, Libertarianism In One Lesson, p. 35 “There is only one fundamental right (all others are its consequences or corollaries): a man’s right to his own life. Life is a process of self-sustaining and self-generated action; the right to life means the right to engage in self-sustaining and self-generated action–which means: the freedom to take all the actions required by the nature of a rational being for the support, the furtherance, the fulfillment and the enjoyment of his own life…Since man has to sustain his life by his own effort, the man who has no right to the product of his effort has no means to sustain his life.” – Ayn Rand, Capitalism: The Unknown Ideal, pp. 321-2 “The right of life and liberty–that is to say, the right of the man to himself–is not really one right and the right of property another right. They are two aspects of the same perception–the right of property being but another side, a differently stated expression, of the right of man to himself. The right of life and liberty, and the right of the individual to himself, presupposes and involves the right of property, which is the exclusive right of the individual to the things his exertion has produced.” – Henry George, A Perplexed Philosopher, p. 210

11.4. Is a tax on land values a real tax? -N

This depends, of course, on what is meant by “tax.” If a tax is a sum of money levied by government on any income or asset, then LVT is a tax. However, if we see taxes as a burden on the production of wealth, we must realize that a levy on land values does not fall in this category. This is because of land’s particular character as an economic factor: it is fixed in supply, and needed for all production. Land rent is paid in any case — if not to the community, then to the private landholder. Compared to other levies that burden production, such as income or sales taxes, LVT has the character of a “user fee” — it is the public appropriation of an economic surplus, and does not burden production at all.

11.5. But taxation is an unnecessary evil.

There is no such thing as an “unnecessary evil” or a “necessary evil” because evil is not well-defined. “Evil” is an illusion.

Moreover, there has never been a single civilization in human history that has managed to exist without some form of taxation. If civilization is to exist, then taxation is inevitable because a society can only eliminate what Libertarians call “force” by exerting it outwards and establishing the Rule of Law within the society.

So if taxation has to exist within a civilization, it’s only logical that the least harmful and most widely recommended tax of them all should be used.

11.6. But it’s “authoritarian” to take away someone’s land if they don’t pay their taxes.

This belief is based on an incorrect understanding of what does and does not count as aggression. An equal right to land is a negative right since land is not created by other people’s labor. It follows that everybody has an equal right to land, which is guaranteed by collecting LVT and using it to benefit the public. Economically, it’s also beneficial because if land is confiscated, then that means that the land is not being used to its maximum economic efficiency to the point where paying the LVT price is not a problem. This is a good thing because then someone who will use the land to its fullest economic potential will have a fair chance to occupy it and use it too.

Georgism would lead to unjust and uncompensated confiscations.

The same could be said for any form of government. There is the possible concern that land values could be appraised in such a way that promotes corruption or cronyism, but this isn’t a very good argument because prices have to be bootstrapped somehow. If anything, implicit ad hoc pricing creates even more opportunities for corruption and evasion, in comparison to taxing natural resources at the point of extraction. If the government doesn’t set prices for natural resources, then markets won’t be bootstrapped, and we’ll have a tragedy of the commons. https://gameofrent.com/content/can-land-be-accurately-assessed

11.7. But it’s authoritarian to disrupt inherited property.

That claim is akin to the Divine Right of Kings, like a royal dynasty’s ownership of land. In order to ensure private ownership of land, you have to prevent other people from using it. That’s pretty authoritarian considering that the inheriters did no labor to create the land.

Whether or not LVT disrupting inheritance is “authoritarian” also depends on the inheritance. To be clear, we’re not against land being possessed by family generations. We’re only saying that they should pay LVT at the market price if they want to keep the land in the family.

Being related by blood is not a qualification for anything. Society should be based on merit. If people want to continue owning land that their ancestors owned, then there’s no reason why they shouldn’t have to pay the market price, just like everyone else.

How does paying the market price show merit though? You can be wealthy and still lack merit.

Because Georgism would eliminate rent-seeking, wealth earned without generating wealth. Georgists believe that people should only be entitled to nothing but the fruits of their labor. Any wealth that people do have, would be earned by the fruits of their labor.

Do Georgists support inheritance? If a parent makes savings for their child’s future, should that instead be taken by society and handed out “equally”?

The difference is that land is gained by preventing other people from using it. If a child inherits money from their ancestors, they’re not preventing anybody else from being able to earn money by the fruits of their labor. You can’t say the same thing about land.

But everyone in a society inherits the land though?

I mean, you could argue that LVT is bad because so much of the middle class’s retirement is dependent on the land speculation of their houses. But that’s not a good argument against LVT when considering the costs that that has for urban planning, pollution, economic efficiency, etc. The harms outweigh the benefits.

11.8. Why do people have an equal right to land?

Main Article: Why Everybody Owns Land Equally.

It doesn’t make sense to believe that land can be ’owned’. The idea that land can be ’owned’ is kind of weird. You didn’t create it and You can’t pick it up and move it like you could any other owned object. Ownership in this sense is a bundle of rights that excludes everyone else and requires either everyone else to consent or the government to keep enforcing. Yeah, you don’t ever get to stop paying for those rights, but if it was otherwise, land ownership would just be about calling dibs on something nobody made and expecting the government to defend your rights in perpetuity, even if you aren’t using it. Like, can I call dibs on the moon and expect NASA or ESA to kick anybody else’s probes and rovers off? There is something fundamentally wrong with applying a Finder’s Keepers System to land rights. Land ownership can be useful useful, but philosophically, it isn’t very different from calling dibs on the moon, and doesn’t make much sense outside of community consenting to your rights in return for compensation.

11.9. Doesn’t the NAP presuppose the OATP?

[I haven’t finished writing this section yet. It takes time to write stuff.]

11.10. But labor doesn’t count as property.

[I haven’t finished writing this section yet. It takes time to write stuff.]

11.11. Wouldn’t the LVT discourage production? -N

No, because the value of land has no reference to a cost of production; it is purely a function of demand. This, among other things, led Adam Smith to conclude that: “Both ground-rents and the ordinary rent of land are a species of revenue which the owner, in many cases, enjoys without any care or attention of his own. Though a part of this revenue should be taken from him in order to defray the expenses of the state, no discouragement will thereby be given to any sort of industry….Ground-rents and the ordinary rent of land are, therefore, perhaps, the species of revenue which can best bear to have a peculiar tax imposed upon them.” [Emphasis mine] – The Wealth of Nations, Bk 5, Ch. 2, Pt 1 Nobel prize-winning economist, Paul A. Samuelson, reached the same conclusion two centuries later:

“The striking result is that a tax on rent will lead to no distortions or economic inefficiencies. Why not? Because a tax on pure economic rent does not change anyone’s behavior. Demanders are unaffected because their price is unchanged. The behavior of suppliers is unaffected because the supply of land is fixed and cannot react. Hence, the economy operates after the tax exactly as it did before the tax–with no distortions or inefficiencies arising as a result of the land tax.” [Emphasis original] – Economics, 16th ed., p. 250

What is even more “striking” is that Samuelson’s remarks are only half-true. Not only will a tax on rent lead to no distortions or economic inefficiencies, it will actually stimulate the economy by (1) lowering the entrance-barrier into the market place, and (2) encouraging much more efficient use of land within that market place. A well-documented case in point is the overall success of the “split rate” property tax (whereby land values are taxed at a higher rate than improvements) in over a dozen localities throughout Pennsylvania. It is the taxation of wages and interest that discourages production – “wages” being the return to labor, and “interest” the return to capital. Thus, it follows that the more we shift the tax burden off labor and capital and onto land values, the more prosperous the economy will be overall. Henry George put it this way:

“To abolish that taxation which, acting and reacting, now hampers every wheel of exchange and presses upon every form of industry, would be like removing an immense weight from a powerful spring. Imbued with fresh energy, production would start into new life, and trade would receive a stimulus which would be felt to the remotest arteries. The present method of taxation… operates upon energy, and industry, and skill, and thrift, like a fine upon those qualities. If I have worked harder and built myself a good house while you have been contented to live in a hovel, the taxgatherer now comes annually to make me pay a penalty for my energy and industry, by taxing me more than you. If I have saved while you wasted, I am mulct, while you are exempt. If a man build a ship we make him pay for his temerity, as though he had done an injury to the state; if a railroad be opened, down comes the tax collector upon it, as though it were a public nuisance; if a manufactory be erected we levy upon it an annual sum which would go far toward making a handsome profit. We say we want capital, but if any one accumulate it, or bring it among us, we charge him for it as though we were giving him a privilege. We punish with a tax the man who covers barren fields with ripening grain, we fine him who puts up machinery, and him who drains a swamp….

“To abolish these taxes would be to lift the whole enormous weight of taxation from productive industry. The needle of the seamstress and the great manufactory; the cart horse and the locomotive; the fishing boat and the steamship; the farmer’s plow and the merchant’s stock, will be alike untaxed….Instead of saying to the producer, as it does now, ‘The more you add to the general wealth the more shall you be taxed!’ the state would say to the producer, ‘Be as industrious, as thrifty, as enterprising as you choose, you shall have your full reward! You shall not be fined for making two blades of grass grow where one grew before; you shall not be taxed for adding to the aggregate wealth.’” – Progress & Poverty, pp. 434-435

11.12. Since people need food to sustain their lives, and since food, like land, is in limited supply, could not the same argument for taxing the value of land be used to justify taxing the value of food? -N

No, because (1) while food is in “limited” supply, it is not in fixed supply; and (2) with food starvation is not the only alternative to purchasing it from others, whereas with land it is. With food, one can always produce instead of buy. Not so with land. Some might counter that one can always produce to earn the wages needed to acquire land, but this presupposes the very issue in question – access to land. While it is true people can always acquire land by earning the wages needed to rent or purchase it, one cannot earn wages to begin with unless one first has access to land, which brings us right back where we started.

Food is a product of labor; land is not. Thus, the notion that one has an exclusive right to the fruits of one’s labor is incompatible with the notion that there is a common right to the value of those fruits, while it is not incompatible with the notion that there is a common right to the value of land.

11.13. Isn’t Georgism the same thing as Socialism or Communism?

No, not in the slightest. A Socialist economy is when Capital is publicly owned, a Georgist economy is when Land is publicly owned while Labor and Capital are privately-owned, and a Communist economy is when all Three of Factors of Production (Labor, Capital, and Land) are publicly owned.1 You can refer to the table for a concise comparison:

  Capitalism Georgism Socialism Communism
Land Private Public Public Public
Capital Private Private Public Public
Labor Private Private Private Public

Note the following for what it means for each Factor of Production to be “public”:

Basically, the distinction between whether each factor of production is public or private is whether it is owned by the state or owned by individual entities.

Unlike Capitalism, Socialism, and Communism, Georgism has never been tried before, and yet it is different from all of them, as it is the only one where everybody is guaranteed equal access to all Three of the Factors of Production.

11.14. But private land ownership is necessary to resolve tragedies of the commons.

This is not true. Private land possession is the only sufficient condition required for solving Tragedies of the Commons. Meanwhile, private land ownership causes the Iron Law of Wages (in combination with population growth), mass poverty, untold economic inefficiency, and is unnecessary for solving TOTCs. Since private land ownership is not required to solve TOTCs, this is not a problem with Georgism.

As an example, think about a company with shareholders. Each of the shareholders owns part of the company, but they don’t necessarily get to make decisions for the company.

Possession and ownership are different concepts. Different words, different wiki articles, different dictionary definitions: Wiktionary: ownership & Wiktionary: possession.

If we got rid of private land ownership and replaced it with Georgism, there would be no practical difference.

This is also wrong. If Georgism was installed today and everybody owned land equally (while private possession is still maintained), then the difference would be that everybody receives a UBI after all government expenses are paid. In essence, everybody would own the Earth’s land equally because the value would be equally distributed to all citizens.

11.15. But land is neither necessary nor sufficient to create economic value.

I suppose that’s why it’s so cheap and can be bought for pennies even in dense urban areas.

Oh, wait…

[I haven’t finished writing this section yet. It takes time to write stuff.]

11.16. Isn’t land less important in today’s economy than it was decades ago?

No. To understand why, simply ask yourself the following question. If the importance of land is indeed going down, why does the price of land keep going up? The answer is that, as the economy grows, the importance of land grows along with it – especially for the working poor. If you doubt this, visit the following links:

11.17. But Capitalism already has a “free market” for land.

There’s nothing free market about some guy calling dibs on a plot of land. Alloidal libertarians are able to recognize that inflation is a form of invisible theft, and yet they turn a blind eye to land speculation which also extracts other people’s labor into the hands of a few, yet differently.

The irony is that they need LVT in order to actually have a free market. The ’free market’ rhetoric is typically used to defend what is essentially modern-day feudalism. And both sides bought into the propaganda. Even the left is convinced that we have a free market, since they just responded by declaring that freedom is bad.

11.18. Shouldn’t there be a free, private market in land, in order to allocate land sites to their most efficient uses? -N

No, because the tax on land value is independent of its actual use, based only on its potential in its highest and best use. In many cases the lack of LVT causes premature development expecting higher land value, and in other cases causes speculators to avoid developing, waiting for higher land values. LVT promotes the optimal timing, because the opportunity cost of not developing is explicit: the tax must be paid regardless of how the site is being used. Without land value taxation, land value is subsidized, distorting incentives.

Critics of LVT claim that rent is earned as landlords actively seek out the best tenants and the best use of a site. But this is not rent; the return on this exertion is wages. Those seeking the best tenants and sites are in the role of entrepreneur, not landlord. Some of the rental that tenants pay is wages to the entrepreneur and to the manager (who could be a different person than the landowner).

11.19. Isn’t concentrated ownership of land just, so long as it’s based on voluntary transactions? -N

No, because if only some people “own” the earth, then only some have a right to live upon it. All individuals must have access to the earth in order to exercise their right to sustain their own lives. Thus, to allow the earth to become the unconditional property of a relative few is to deny this right to everyone else, since it makes the latter obligated at birth to pay the former for mere access to the planet – as if the former were responsible for the earth’s very existence. While the private collection of land rent may seem harmless at a micro-level, at a macro-level it constitutes an entitlement scheme, whereby Group A receives payment from Group B, even though Group A renders no service in return. In that sense, it violates the right of the members of Group B to the fruits of their labors. “As soon as the land of any country has all become private property, the landlords, like all other men, love to reap where they never sowed.” – Adam Smith, The Wealth of Nations, Bk 1, Ch. 6 If some people fail to see this, it is because they, in the words of Henry Hazlitt, “overlook the woods in their precise and minute examination of particular trees.” In this case they overlook the affect that private rent-collection has on the economy as a whole in their precise and minute examination of particular transactions, and how these transactions benefit particular groups. Overall, the payment of land rent to the few at the expense of the many imposes on the latter artificially high costs of living on one hand, and artificially low wages on the other. To learn more about why the current land market is anything but “voluntary,” read the following article by Fred Foldvary. http://www.progress.org/archive/fold239.htm

11.20. As a general rule, taxation is wrong since it involves the use of force. Is a “tax” on land value an exception to this? -N

Yes, for the simple reason that “force,” as such, is neither good nor bad. If used to defend one’s person or property from aggressors, or to enforce payment of a rightful debt, it is a good thing. If used to harm the person or property of a non-consenting other, or to enforce payment of a wrongful debt, it is a bad thing.

A tax on wages or interest implies that the income one receives in return for the exertion of one’s labor, or for the use of one’s capital goods, belongs (at least in part) to others. This conflicts with the basic libertarian principle that you have an exclusive right to the fruits of your labor.

A tax on rent implies that the income one receives for the value of the land one holds belongs to others. Since land itself (1) is not the fruit of anyone’s labor, and (2) is that to which all have an equal right of access; and since the rent of land (1) is not a return to labor, and (2) reflects the extent to which Locke’s proviso has been violated, a “tax” on rent does not conflict with the principle that you have an exclusive right to the fruits of your labor, but is in fact a just and necessary means of upholding it.

Thus, the part of one’s income that is taken via taxation of wages and interest constitutes the enforcement of a wrongful debt, whereas the part of one’s income that is taken via taxation of rent constitutes the enforcement of a rightful debt.

“As to what constitutes robbery, it is…the taking or withholding from another of that which rightfully belongs to him. That which rightfully belongs to him, be it observed, not that which legally belongs to him.” [Emphasis original] – Henry George, Property In Land, p. 46.

Still, critics will argue, a tax on rent involves the use of force, and is therefore wrong. The problem with this argument becomes evident when they are presented with the scenario of a tenant no longer able to pay a titleholder for the value of the land he is using, and then asked whether or not it would be legitimate to use force to remove the tenant from the titleholder’s land. They typically answer yes to this question, and when pressed for an explanation, finally concede that yes, thereis such a thing as a legitimate use of force when it comes to upholding a rightful debt.

The dispute, then, is not over whether force, in and of itself, is right or wrong, but whether the debt in question is right or wrong – i.e., whether or not the taxation of rent conflicts with the libertarian principle that each person has property in himself and, by extension, in the fruits of his labor. Geolibertarians hold that it does not so conflict, since rent, as mentioned before, is not a return to labor. Rent is in fact a return to land, meaning the percentage of one’s income one could receive simply by renting out the land one holds to someone else. Yet to whom does this value rightfully belong? Since land values derive, not from what titleholders do, but from the extent to which “others” (particularly those who make up the surrounding “community”) are denied access to land they wish to use, and to which they have an equal right of access, it follows that this value is rightfully owed to these others, while wrongfully owed to titleholders. All individuals have an equal right to land, so all have an equal right to the rental value thereof.

11.21. Didn’t Austrian economist Murray Rothbard refute the LVT? -N

No, but not for lack of trying. Rothbard’s argument against the LVT is fatally flawed for at least two reasons – one moral, the other economic. From a moral perspective, it completely ignores the unjust interference that the overextension of law-made property imposes on man-made property. From an economic perspective, it is based on a false understanding of what conditions are necessary for land to have rental value. In Libertarian Party at Sea on Land, LP activist Dr. Harold Kyriazi explains why Rothbard’s attack on the LVT was misguided at best. The following is from pages 57-61 of that book:

The only well-known libertarian writer whom I know to have explicitly, and at great length, opposed the idea of community collected user fees for natural resources is Murray Rothbard, which is odd, given his admiration for Albert Jay Nock and Frank Chodorov, who, in turn, revered Henry George. Rothbard apparently had extensive discussions with Georgists:

If every man owns his own person and therefore his own labor, and if by extension he owns whatever property he has “created” or gathered out of the previously unused, unowned “state of nature,” then what of the last great question: the right to own or control the earth itself? … It is at this point that Henry George and his followers, who have gone all the way so far with the libertarians, leave the track and deny the individual right to own the piece of land itself, the ground on which these activities have taken place. (pp. 33-34, For a New Liberty.)

The following is taken from his The Ethics of Liberty.

(p. 50, footnote 2): A modified variant of this “Columbus Complex” holds that the first discoverer of a new island or continent could properly lay claim to the entire continent by himself walking around it (or hiring others to do so), and thereby laying out a boundary for the area. In our view, however, their claim would still be no more than to the boundary itself, and not to any of the land within it, for only the boundary will have been transformed and used by man.

With this statement, Rothbard may seem to have carried the “first use” doctrine to its illogical extreme. (If walking over some land constitutes transformation and use, then is it just one’s footprints that one owns? Or does one’s rightful claim extend out to all the underbrush one has cleared away? Or, can one claim land as far as the eye can see? This is the very definition of the word “arbitrary.”) But in his defense, to convert the claim into actual ownership would, Rothbard would say, require actual use (though we’re again faced with the question of what constitutes “use” – see p. 79, “Anti-Rothbard…”). For example, earlier, in a Robinson Crusoe paradigm, he stated that Crusoe’s “true property–his actual control over material goods–would extend only so far as his actual labor brought them into production. His true ownership could not extend beyond the power of his own reach.”

What, then, would Rothbard say about large American corporations owning, but not using, millions of acres of land, as some now do? He gives us his answer in an essay he wrote on Henry George’s Land Value Tax idea, entitled “The Single Tax: Economic and Moral Implications” (FEE “Special Essay Series,” 1957). Here are a few examples from that work: Well, what about idle land? Should the sight of it alarm us? On the contrary, we should thank our stars for one of the great economic facts of nature: that labor is scarce relative to land…Since labor is scarce relative to land, and much land must therefore remain idle, any attempt to force all land into production would bring economic disaster. Forcing all land into use would take labor and capital away from more productive uses, and compel their wasteful employment on land, a disservice to consumers. [Emphasis Rothbard’s.]

Of course, LVT would and could do no such thing, as those who strive to put idle land into productive use would have to bid against other land users for labor, and only the best uses of labor and land would win out. Thus, rather than forcing all land into use, LVT would discourage all but the most productive use of land, just as any market tends to allocate resources most wisely. Another thing that would happen is that the earnings of labor would increase due to increased competition for it, and (ideally) none of the produced wealth would go to landowners qua landowners. Let me rephrase Rothbard’s last sentence in a way that makes sense: Forcing land users to pass over ideal idle land and utilize marginal land instead, is wasteful of human labor and natural opportunities, a disservice to all mankind and a boon only to landlords and land speculators.

But here’s the most embarrassing passage:

A 100% tax on rent would cause the capital value of all land to fall promptly to zero.

Correct.

Since owners could not obtain any net rent, the sites would become valueless on the market.

False! They’d be valueless only to those market participants who wish only to speculate in land, not to those who wish to use land in some productive endeavor.

From that point on, sites, in short, would be free.

Wrong again. While it’s true there’d be no sale price for vacant land, one would still have to pay the ground-rent to use it.

Further, since all rent would be siphoned off to the government, there would be no incentive for owners to charge any rent at all.

Wrong yet again. He’s assuming the LVT would be set by an actual ground-rent charged by the landlord, rather than being an assessed value that would have to be recouped. And, I might add, total rental costs would tend to decrease as additional units come on the market as the monopoly stranglehold on land loses its grip.

Rent would be zero as well, and rentals would thus be free.

He continues to pound a straw man.

The first consequence of the single tax, then, is that no revenue would accrue from it.

He took a wrong turn, and just keeps going!

Far from supplying all the revenue of government, the single tax would yield no revenue at all! For if rents are zero, a 100% tax on rents will also yield nothing.

Rothbard then goes on to state,

Compelling any economic goods to be free wreaks economic havoc…the result is to introduce complete chaos in land sites.

Completely false. Even if LVT were applied at a national level, and there were no competition among municipalities for residents, people would still bid on the leases of occupied property, providing price information. (For more on this, see p. 97, “How would LVT work?”) In Power and Market: Government and the Economy (second edition, 1977), Rothbard went even further into the realm of irrationality in his attempt to refute Georgist land theory (p. 131):

Contrary to Georgist doctrine, however, the land problem does not stem from free-market ownership of ground land.

I know of no Georgist who would ever use the phrase “free-market” in conjunction with our current, individual monopoly market in land.

It stems from failure to live up to a prime condition of free-market property rights, namely, that new, unowned land be first owned by its first user, and that from then on, it become the full private property of the first user or those who receive or buy the land from him.

It is an obvious fiction that any use, however small or large the effort, should grant full private ownership for all time, unless we’re talking about a make-believe world with unlimited land where access to all of it is instantaneous (i.e. where travel time is zero). This fiction ignores the fact that someone who, for example, puts up a fence and lets a cow graze, is much less the rightful “owner” of land than one who builds an industrial plant or a shopping mall. (For more on this, see p. 79, “Anti-Rothbard…”)

11.22. Isn’t the LVT based on the Marxist idea that the right to land is a collective right? -N

No, it is based on the Lockean idea that the right to land is an equal right. By that I mean: the idea that an individual has “property” in land only to the extent that there is, in the words of John Locke, “enough, and as good left in common for others.” In that sense, the right to land is not a collective right, but anindividual right that exists independently of the collective (i.e. “society”). The equality of this right is merely a limitation that arises from the presence of others with like rights. By contrast, a collective right to land dictates that an individual does not have a right to use any land unless society – either explicitly or by omission – has granted him the right to do so. With the equal right to land, one does not require the consent of society to use land. The right to the use of land belongs at birth to each individual. So while the consent of others is not needed, it is, nevertheless, necessary that in the exercise of that right, one does not infringe upon the equal right of others – i.e. violate Locke’s proviso that there be “enough, and as good left in common for others.” And since the rental value of land provides an accurate measure of the extent to which said proviso has been violated, “others” should be compensated in accordance with that value. At the same time, of course, all taxes on labor and capital should be abolished, since they violate the exclusive right that each individual has to the fruits of his own labor.

11.23. What’s wrong with the Homesteading Principle?

Private land ownership violates the NAP.

[I haven’t finished writing this section yet. It takes time to write stuff.]

12. More Specific Questions

12.1. Wouldn’t a tax on natural resources make our country’s natural resources more expensive and uncompetitive with world markets?

No, because this question assumes that natural resources would be privately owned and that the Natural Resources Tax would go on top of that, which is not the case at all under Georgism.

We already pay enormous corporations like British Petrol, De Beers, Sibanye Stillwater Limited, etc money when we buy natural resources from them. But under Georgism, natural resources would be owned by the government instead, and everybody would pay money (Natural Resources Tax) to the government to buy said natural resources, not the privately owned mega-corporations that shouldn’t exist. Since the government would collect the revenue from selling natural resources instead of private corporations and no additional expenses would be added on top of that, Georgism would not make the country’s natural resources less competitive on the global market.

Another point is that this would be even more impossible if there is a global government that manages the world’s natural resources. The whole point of government is to resolve prisoner dilemmas and tragedies of the commons, so it is entirely reasonable to have a global government. The only criticisms that could be applied against a global government are ones that would also apply to national governments as well.

If we don’t build a global government, a de facto global government will take form instead. Today, that is currently the United Nations, combined with the influence of the Chinese and American superpowers. So we might as well build a global government that is less likely to be a dumpster fire like the UN.

It is unreasonable to allow mega-corporations to profit billions of dollars just because they claimed that land first under a finders’ keepers system. That is called rent-seeking, and it doesn’t contribute to economic growth at all. If anybody is going to be making billions of dollars off of selling coal, oil, iron, copper, cobalt, natural gas, etc, it better be the government so that everybody else can pay fewer taxes (and maybe receive some welfare in return).

12.2. What makes Location Value Taxation unique?

It is the only natural resource that all countries and regions of the world, have in common. Not every country will have coal, oil, gold, timber, etc, but every country will have communities whose economic productivity and urban layouts can be most effectively optimized by location value tax, since it enforces the Space Utility Optimization Principle.

12.3. Does Georgist Theory claim that an LVT is even better than having no taxes at all?

12.4. Do all Georgists support UBI?

No. Other Georgists have all sorts of ideas about how they think tax revenues should be used, but all of them are pretty arbitrary in comparison to the Citizen’s Dividend, which I personally don’t support.

12.5. What happens if the LVT increases, and the land possessor is unable to pay it?

If someone is at risk of this happening to them, then they ought to purchase some insurance that will pay the difference between what they were paying and what they have to pay after the tax increase, in case their LVT rates rise.

12.6. Who pays the LVT when the land title or tenure is uncertain?

If it’s useful to keep the land as public land (i.e. doing so raises neighboring rents by more than the rent on that land), then that’s what we would do.

Otherwise, we’d make the title certain by drawing up an appropriate contract. If nobody already claiming the land steps forward to pay the going rate, or to ask compensation for any improvements, then whoever signs the contract and pays the LVT becomes the new tenant (and the value of the improvements gets incorporated into the LVT rate).

12.7. How do you sell the improvements when someone else repossesses the land?

[I haven’t finished writing this section yet. It takes time to write stuff.]

See: How would the value of improvements to the land be separated from the land value itself?.

Read More: General Single Measurements For Multiple Variables.

12.8. Why do Internet domain names also count as land?

[I haven’t finished writing this section yet. It takes time to write stuff.]

12.9. What do Georgists think about Net Neutrality?

[I haven’t finished writing this section yet. It takes time to write stuff.]

13. Glossary -N

Arbitrage
The ability to buy cheap and sell for a higher price. Little productive effort goes into the act.
Capital Improved Rating
A system of municipal rating that taxes a portion of the improvements (the building) as well as the land. This is a tax on progress and is a subtle subsidy for land speculation.
Classical economists
The economic era defined by the work of the 18th century Physiocrats, through to Adam Smith, Ricardo, JS Mill, Karl Marx and Henry George. They defined output as reliant upon three factors of production – land, labor and capital.
Neo-Classical economics
The economic analysis defined by the factors of production as labor and capital, with land (representing the earth) a subset of capital. All output is produced by capital and labor.
Commons
Gifts of nature to all living beings that are held in common, rather than privately owned. This concept has expanded from the land, the air, the water to include the digital commons, public libraries and our DNA code.
Common Wealth
Monetary and non-monetary value of the commons in supporting life and well-being. Like stockholders’ equity in a corporation, it may increase or decrease from year to year depending on how well the commons is managed.
Compliance costs
The burden of filling out tax paperwork and complying with regulation.
Deadweight costs
The loss to society caused by market distortions that see less product supplied to market at a higher cost. Usually due to taxes that can be passed on.
Economic Rent
Originally explained by economist David Ricardo (Ricardian rent) as the excess return to some agricultural land over and above the same effort on the least productive land. i.e. productivity returns on land near a river compared to the marginal desert-like land.
  • JS Mill called the excess return from the same effort the “unearned increment”.
  • The term economic rent has been expanded to include all unearned income from ownership of a resource, from a monopoly, from natural scarcity, or any other reason resulting in unearned excess profits not due to work, risk or enterprise. It is also defined as the excess revenue over and above what it takes for a business to reap normal profits. This is the origin of the derogative term “rent-seeking”, referring to people who reap where they did not sow.
  • A simple example of economic rent are oil prices. It has been estimated that oil from the most expensive wells in deep ocean water cost about $60 per barrel to extract including all other costs and normal profit. Easier-to-extract oil costs much less. At the 2008 price of $147 dollar per barrel, oil companies received economic rent of at least $87 per barrel on deepwater wells. The source of their “windfall profits” is economic rent.
FIRE sector
An acronym for Finance, Insurance and Real Estate, combined in the US national income accounts to reflect the symbiosis between these sectors. Michael Hudson has identified as the three sectors of the economy to benefit most from unearned income.
  • The essence of classical political economy was that no outlay of living or embodied labor is needed to obtain rent and interest. This analysis offended the vested interests. The ensuing marginal utility theory ignored the wealth addiction that historically has gone hand in hand with rentiers and the tendency for their compound interest demands to approach infinity.
Georgism
The belief that the fruits of the earth can finance government in a simple, sustainable and efficient manner. Based on the teachings of Henry George, it recognises those who hold ‘ownership’ of the earth and natural monopolies have an advantage over the rest of society. The tax system is used to rebalance the opportunities between land, labour and capital.
Geoists
A name for progressive Georgists (alongside Geonomics) who support taxes on the wider commons including carbon, privatisation and money creation.
Land Banking
The practice of hoarding land as a form of saving ‘in the bank’, in the understanding land prices will increase with the development of society and the increase in population.
Land Tax
An annual charge based on a set percentage of the valuation of land. With land naturally rising in value according to population and technology developments, many economists see it as a natural source of government revenue. Land must be valued annually.
Land Value Tax
Similar to land tax, it merely recognises that land prices will fall back to what can actually be earnt from a location, reflecting its value. Land price includes bubble-like pressures on price. Land values are calculated by property valuers for municipal rates.
  • Generally speaking, property valuers are more conservative in valuations than surrounding property sales evidence may indicate, due to their preference for land values over prices.
Monopoly
The market dominance of an industry by a single entity. In this website, the term is used widely, sometimes in place of the more cumbersome ‘oligopoly’. Oligopoly infers the market control of an industry by a small number of firms.
Negative Gearing
is a form of financial leverage where an investor borrows money to invest but the gross income generated by the investment is less than the cost of owning and managing the investment, including interest charged on the borrowings. i.e. purchasing a house at above market price in the knowledge the difference between the possible rental income earnt and the mortgage payments required can be written off using negative gearing. A top income earner would therefore receive a 45% tax subsidy on the loss making portion of that equation. Future expected capital gains are planned to make up any other cash flow loss.
Property Rights
Private property is sacrosanct. However a clear distinction must be made between that which is man made and that which was a gift to all living beings. Georgists believe land should be leased to the state in recognition that no human produced the land, the earth. The improvements (the house) are to be privately owned. Both the lease and improvements can be sold at auction.
  • This distinction between man made and gifts of the creator is enhanced by John Locke’s classical theory of property, which proposes an individual has the full right to one’s labour of goods they produced. Further clarification is given by the Lockean proviso – a feature of John Locke’s labour theory of property which says that, whilst individuals have a right to homestead private property from nature by working on it, they can do so only “…at least where there is enough, and as good, left in common for others”.
Rent Seeking
The process of wealth accrual without any productive output. This results from lobbying for monopoly powers via insider contacts. Rent seeking results in poor resource allocation, lower economic output, lower government revenues, increased inequality, higher cost-of-business and national decline. Adam Smith called rent seekers ‘the public enemy’.
Resource Rents
Economic rent in relationship to subsoil minerals, oils and gases. Identifying and measuring (or collecting) resource rent depends on the availability of information, market conditions, technology and the system of property rights used to govern access to and management of resources.
Sales Tax
A percentage charge on the cost of goods purchased. Known as the GST (Australia), VAT (EU), Sales Tax (US).
Site Value Rating
A form of council rating where the valuation is based solely on the land component. This focuses on the locational value of a site and ensures the economic rents are all that are collected. Further reading.
Stamp Duty
A form of sales tax paid at the point of a transaction, paid by the buyer. Commonly referenced in respect to housing.
Torrens Title
Arguably world’s best practice, this is the system of land holding registrations maintained by the state. It guarantees an indefeasible title to those included in the register. Land ownership is transferred through registration of title instead of using deeds. Its main purpose is to simplify land transactions and to certify to the ownership of an absolute title to realty.
  • The system was designed by Robert Richard Torrens for the South Australian land title registry when he was appointed Registrar of that state in the 1850’s. He based his simple system on the method of insuring shipping used by Lloyds of London. It used a single register for each land holding and recorded all details and interests affecting that land such as: easements, covenants, mortgages, resumptions, caveats, subsequent changes in ownership.
  • The greatest advantage of Torrens title is that it is a single document guaranteed by the relevant Government.
Unearned Income
The ability to earn income above the margin of production. Investments in real estate are most pertinent, with the supply of land provided as a gift of nature. The cost of production is zero. As houses depreciate and purchasers compete for ‘location, location’, land price increases have been deemed unearned incomes since the Classical Economics era.

Footnotes:

1

This is an over-simplified description to an extent, but it’s still accurate if we’re classifying economic systems purely on whether the three factors of production are publicly or privately owned.

Last Modified: 2024 May 08, 17:07

Author: Zero Contradictions